Categories
Yale

Yale. History of graduate education up to 1898.

The following sketch of the history of Yale’s graduate school was published in 1898. What would become the graduate “seminaries” of the respective disciplines were organized as extracurricular “clubs”. The introduction of graduate fellowships and scholarships is of interest as in the early openness Yale showed with respect to graduate admission for women. Arthur Twining Hadley enters the Yale scene as professor of political science and later of political economy as well as serving as the Graduate Dean.

_______________________________

[343]

CHAPTER VI
The Graduate School

GRADUATE instruction, apart from that leading to one of the three “learned professions,” [i.e. clergy, law and medicine] was probably not thought of at Yale before the present century. Its beginnings can perhaps be traced in the comprehensive plans of President Dwight, who, as one of the founders of the Connecticut Academy of Arts and Sciences in 1799, showed his desire to encourage independent research, and the acquisition of knowledge in other fields than those which had hitherto been almost exclusively cultivated. “Resident Graduates” came here for study during his term of office, but it is not known to what extent their studies were other than theological. The catalogue of 1814, contains the names of seventeen, the first official record of their presence, and the list is continued in succeeding catalogues, rising in one year to thirty-one, until 1824, when it suddenly disappears. But in that year, “Theological Students” are entered for the first time, and the presumption is that they are simply the “Resident Graduates” appearing now under their proper descriptive title. In1826, appear the names of four Resident Graduates, and as the students of the three professional Schools are all separately entered, we doubtless have here the first reliable list of non-professional graduate students. Three of these were Bachelors of Arts from Amherst College, one being Charles U. Shephard, afterward for many years a Professor at Amherst, and well known for his mineralogical collections. For the next twenty years, with a few exceptions, lists of graduate students appear in the Catalogue, the largest number for anyone year being seven. Among these were Robert McEwen and Gordon Hall, afterward prominent clergymen; B. G. Northrup, the well-known Superintendent of Education in Connecticut; Noah Porter, James D. Dana and Denison Olmsted, Professors at Yale; and William L. Kingsley, for many years Editor of the “New Englander.” The instruction of this class of students is known to have appealed especially to the scholarly enthusiasm of President Woolsey during the years of his Professorship, and their claims always received his special attention. Professor Thacher also, with his usual forethought, expressed at an early date his desire that provision might be made for them.

In 1841, an important step was taken in the appointment of Edward E. Salisbury as Professor of Arabic and Sanskrit. This was the first provision made for the instruction of graduate students by other than College Professors whose attention was mainly given to undergraduates. It was also the first recognition in this country (if the importance of Sanskrit in the study of language, and, so far as demand for instruction went, was in advance of the time. For eight years no student presented himself; then two came. They were William D. Whitney and James Hadley. The former had taken his first degree at Williams College, and came to Yale for graduate study, attracted by Professor Salisbury, who was the only Professor of Sanskrit in the country. He studied here one year, in 1849-50, then went to Germany for three years. He returned to Yale in 1854, and took the Chair of Sanskrit which had been vacated for him by Professor Salisbury, who retained the Chair of Arabic two years longer.

Professor Whitney’s appointment came at a time when the Graduate School was beginning to emerge clearly to view as a distinct section of the new Department of Philosophy [344] and the Arts. This, as is elsewhere stated, commenced in 1847, and was opened to “graduates and others.” That year there were eleven students, five of whom were undergraduates. Contrary to expectation, the number of the latter greatly increased, so that in 1852, it was found best to classify them in separate Schools of Chemistry and Engineering, leaving two graduates who were not pursuing those studies. These were Daniel C. Gilman and Hubert A. Newton. In 1854, the year of Professor Whitney’s appointment, the courses in Chemistry and Engineering were brought together under the title “Yale Scientific School,” and the following year a scheme of lectures and instruction designed especially for graduates not in the Scientific School appears.

In 1861. the degree of Doctor of Philosophy was conferred for the first time, and its recipients were Eugene Schuyler, James M. Whiton and Arthur W. Wright. These three scholars, since so well known in their respective lines of work, were, so far as academic form goes, the first finished product of the Yale Graduate School. Yale was the first institution in the United States to confer this degree on the basis of at least two years’ resident graduate work, with a final examination and thesis giving evidence of high attainment. It furnished to young men of ability and ambition, but moderate means, the opportunity to earn this most highly prized of all academic degrees without going abroad, and at the same time gave a notable impulse to the cause of advanced scholarship in the United States.

The award of the degree in 1861, gave consistency and dignity to the courses leading to it, though much remained to be done in the way of development and further organization of a Graduate School. In 1872, the Department of Philosophy and the Arts was re-organized, as elsewhere mentioned, so as to include all the sub-departments of instruction outside the three Professional Schools, and the graduate students, both of letters and science, in the new Department, were entered in a single list in the Catalogue. At the same time the Graduate School was given a definite organization by the appointment of an Executive Committee to “receive and record the names of applicants for instruction, and judge and approve the courses of study proposed.” Shortly after, the number of degrees to be awarded in the School was increased. These at first were Doctor of Philosophy and Civil Engineer. In 1873, that of Mechanical Engineer was added. In 1874, the degree of Master of Arts, hitherto given in course to Bachelors three years after graduation on payment of five dollars, was rescued from its comparative worthlessness as a certificate of longevity and pecuniary ability, and was made to depend upon one year of non-professional study. In 1897, the degree of Master of Science was established.

In 1892, the organization of the School was further improved by the appointment of Professor A. T. Hadley as Dean. At the same time a step of much significance was taken, in the opening of the School to the graduates of Women’s Colleges, who were invited to come here and study for the degree of Doctor of Philosophy. This practical recognition of the needs of women, and of their right to participate in the advantages of the more highly specialized courses to be found only at the larger Universities, was accorded to them in New England first at Yale. This move was received with much interest in academic circles, and has met with a fair measure of success. The matter of pecuniary assistance, combined with honorable recognition of merit, was also taken up. Five fellowships of $400 each, and twenty scholarships of $100 each, were established by the Corporation. These were to be open to all members of the School, though the fellowships   were to be given by preference to students in their second year who had shown marked ability in the first. In 1895, Professor [345] Phillips succeeded Professor Hadley as Dean, and was established in a convenient office where he zealously looks after the interests of the School. In 1896-7, its membership was two hundred and twenty-seven, including thirty-one women, an increase of fourfold in ten years.

The Faculty of the School consists of the Professors of the four sub-departments of the Department of Philosophy and the Arts, with Lecturers and Instructors wherever available, and University Professors whose time is given mostly to research. The latter have been few in number, owing to the very limited resources of the University. One of Yale’s greatest needs to-day is large endowment for University Professorships which will furnish opportunities for lives devoted to the highest work of the scholar, such as are hardly possible when time and strength are mainly given to undergraduate teaching. It is no disparagement of the work of the teacher to say that in practice it is apt to interfere with the best work of the scholar. Both are necessary to the highest usefulness of a University, but in the assignment of work, the best results can be obtained by a judicious release of some from undergraduate teaching, rather than by the requirement of substantially the same amount from all. The University Professorship furnishes the golden opportunity for advancing the bounds of knowledge along scholarly lines.

Mention has been made of the appointments of Professors Salisbury and Whitney. In 1866, Othniel C. Marsh was appointed Professor of Palaeontology. His work has been done mainly in connection with the Peabody Museum. In 1871, Josiah W. Gibbs was appointed Professor of Mathematical Physics. In 1877, Samuel Wells Williams, the well-known and eminent student of Chinese language and history, accepted a Professorship of Chinese, which he kept until his death in 1884. In 1886, William R. Harper came as Professor of Semitic Languages, and Arthur T. Hadley was appointed Professor of Political Science. These appointments awakened much interest, and the membership of the School was nearly doubled in five years. At the end of that time Professor Harper left to assume the duties of President of the Chicago University, and Professor Hadley was transferred to the Chair of Political Economy in the College. In 1895, Edward W. Hopkins was appointed Professor of Sanskrit to succeed Professor Whitney, who died in 1894.

From the date last given it will be seen that Professor Whitney was connected with the Graduate School for forty years, which is substantially the whole period of its existence. In a certain sense he was the gift of Professor Salisbury to Yale. It was Professor Salisbury who as his teacher in 1850, discovered his special gifts and encouraged him to cultivate them, then in 1854, made a place for him by giving up to him a portion of his own work, and again in 1869, made it possible for him to remain here by endowing for him the Chair of Sanskrit and Comparative Philology. In that year President Eliot signalized the first month of his Presidency by inviting Professor Whitney to Harvard, and the latter would have felt constrained by financial considerations involving the welfare of his family to accept, had it not been for the prompt and generous action of his former teacher and life-long friend. Concerning this invitation Professor Lanman of Harvard has said, “It reflects no less credit upon Mr. Eliot’s discernment of character and attainments than upon Mr. Whitney’s surpassing gifts, that the youthful President should turn to him, among the first, for aid in helping to begin the great work of transforming the Provincial College into a National University.” Professor Whitney gladly remained at Yale and made it a centre of Philological study for the country. Of his work here Dr. [346] Ward of the “Independent” has said, “What Harvard did for the science of life in America through Agassiz, Yale did for Indo-European philology through Whitney.”

Important agencies in carrying on the work of the School are the clubs, of which there are now eleven, namely, the Classical, Mathematical, Political Science, Philosophical, Semitic, Biblical, Comparative Religion, Modern Language, English, Physics Journal and Engineers, Clubs. The older ones are in a measure revivals of earlier organizations for the promotion of original research; but in their present form they have appeared within the past twenty years, and most of them quite recently. Their membership consists of the instructors and graduate students in the department of study indicated by the name of the club. Their meetings furnish opportunities for interchange of views between teachers and pupils, and thus supplement in a most useful way the more formal instruction of the class-room. In the language clubs, authors are read and discussed. In nearly all, papers are presented which embody the results of individual investigations, and the most important of these have been subsequently read before various larger organizations and printed in their transactions. The Physics Journal Club does not aim at research, but has for its object the reading and discussion of the various periodicals in the field of Physics. Several of the clubs have rooms set apart for their use, and the Classical Club is especially favored in having a commodious, well-lighted room, and a good working-library of its own. For some years it occupied the upper story of the “Old Chapel ;” but when Phelps Hall was completed, it moved into the top story of that beautiful building, where it enjoys its present quarters, exceptionally well arranged and located for quiet uninterrupted work. The opening of the club-room in 1896 was observed with public exercises in the Chapel, where an address was delivered by Professor Gildersleeve of Johns Hopkins University, followed by a social gathering of classical scholars from different parts of the country. During the evening, announcement was made that Mr. Sears of Boston had purchased and presented to the University the valuable classical library of the late Ernst Curtius, the distinguished historian of Greece. This had been pronounced by competent authority in Berlin, “the most valuable library in its department which had been offered for sale in Germany since 1870.” A considerable part of this choice collection of books was placed on the shelves of the Classical Club, where they “increase in a marked degree the facilities for advanced work in the classics.”

A part of the work of the Graduate School is done in connection with the American Classical School at Athens. The Soldiers’ Memorial Fellowship at Yale is conferred upon a Yale graduate who has shown special proficiency in Greek. It may be held five years, and a part or all of that time may be spent at the School in Athens. During ten of the fifteen years since the School started, Yale has been represented by six Soldiers’ Memorial Fellows. Four other Yale men have studied there, so that out of the seventy-three students going from the twenty-three Colleges co-operating in the support of the School, ten have gone from Yale, a number exceeded by Harvard alone. Four of the Directors also, including Professor Richardson, the present head of the School, have been graduates of Yale, which from the first has been one of the most active promoters of the enterprise. A similar school for Latin classical study has been started at Rome, and Professor Peck of Yale is to serve as its Director during the year 1898-9.

The Graduate School claims to be non-professional. This claim rests partly on the fact that the School does not train its students for one of the three traditional “learned [347] professions.” It also rests partly on the theory that the School seeks to promote culture, to strengthen scholarly habits of life and thought, and to widen the fields of knowledge, quite apart from any use which may be made of these acquisitions as capital in the ordinary work of life. It is earnestly hoped that this ideal may be realized in future years, when a goodly number of young men and women may be able and willing to lengthen the period given to a general education before commencing special preparation for a particular calling. At present, however, the School is in fact largely a professional one, furnishing such an equipment as is most useful to the teacher. Its great academic prize, the Ph.D. degree, is sought mainly by those who expect to teach, and is valued largely because it helps its possessor to secure a College Professorship. Such being the case, attention is naturally called to the success of a School in fitting its students for the higher walks of the teacher’s calling, and in this respect the record of the Yale Graduate School is a most honorable one. In the Chicago University, out of fifty-nine Doctors of Philosophy on the Faculty above the grade of Instructor, eleven received their degree from Yale, a larger number than from any other institution, Harvard coming next with six. In all, over one hundred and thirty Professors in different Colleges and Universities have studied at the Yale Graduate School since 1860, but not all have completed the course for a degree. They are widely distributed in the United States, the British Provinces and Japan.

During the past ten years, a number of graduates of the Swedish Colleges, Augustana and Gustavus Adolphus, have been to Yale for their Doctor’s degree. The movement of these Swedes to Yale, especially in view of the fact that most of them have specialized in Philosophy and Biblical studies, has signified more than the individual preferences of the persons concerned. It has been from the first the subject of much interest and careful deliberation in the Swedish Lutheran body in the United States, and the confidence thus shown in the University opens for the latter a most promising and important field of usefulness.

In Japan the name and work of Yale are well-known through the gifted men who have come here for study, mainly in the Law and Graduate Schools, and on returning to their own country have occupied high positions in political and educational life. An interesting episode in the relations of Yale to educational work in Japan was the threefold invitation extended to Professor Ladd by the Trustees of the Doshisha, the teachers of the summer school at Hakone, and certain gentlemen of Tokio who were interested in education. Complying with this invitation, Professor Ladd spent the summer of 1892 in Japan, delivering lectures on Philosophy, especially the Philosophy of Religion. His reception was most cordial, and his lectures, given three times in as many places, were well received by large and attentive audiences. One result of his visit was additional interest in Yale, and desire to secure its advantages, which have brought an increased attendance of Japanese students. It is safe to say that, of American Universities, Yale occupies at present the first position of influence in Japan, and it seems reasonable to believe that the years spent here by men now in influential positions in that country have helped to prepare the way for the liberal policy of the Empire which throws open to Christians the highest offices in the State. Nor, in the matter of maintaining peaceful and friendly relations between the United States and Japan, can it be a matter of indifference that scholarly men of the two countries have worked together, and have learned to respect and trust each other.

 

Source: University and their Sons. History, Influence and Characteristics of American Universities with Biographical Sketches and Portraits of Alumni and Recipients of Honorary Degrees. Editor-in-chief, General Joshua L. Chamberlain, LL.D. Boston: R. Herdon Company. Vol 1 (1898)

Image Source: Arthur Twining Hadley.  Ibid, Vol 2, p. 562.

 

 

Categories
Chicago Suggested Reading Syllabus

Chicago. Economics 300A. Core Theory. Gary Becker, 1956

The first required course in economic theory in a graduate program is intended (to mix metaphors) to get students on the same page and up to speed with core theory. I have posted earlier Chicago material for Jacob Viner, Milton Friedman [ (1946), (1947) and (1948)], and Lloyd Metzler. While this posting moves us well into the 1950s  and out my historian’s comfort zone (i.e. into my own lifetime), for younger followers of Economics in the Rear-view Mirror 1956 probably seems ancient enough to be included here. Besides which, it is never in bad taste to extend a time series by adding an additional observation. It is interesting to note that Alfred Marshall has survived at least this long on the Chicago required reading list for economic theory.

_______________

Economics 300A
Autumn 1956
Reading Assignments by G. Becker

 

NOTES:

1) A knowledge of the material in George Stigler, A Theory of Price or in Kenneth Boulding, Economic Analysis, is a prerequisite.
2) Readings marked with an asterisk (*) are recommended, not required.

 

I. INTRODUCTION

Friedman, M., Lecture Notes, pp. 1-16.
Knight, F. H., The Economic Organization, pp. 1-37.
Friedman, Milton, “The New Methodology of Positive Economics,” in Essays in Positive Economics.
*Hayek, F. A., “The Use of Knowledge in Society,” American Economic Review, September 1945, reprinted in Individualism and Economic Order.
*Keynes, J. N., The Scope and Method of Political Economy, pp. 1-83.

 

II. DEMAND ANALYSIS

Marshall, A., Principles of Economics, Book III, chs. 2-4; Book V, chs. 1-2.
Friedman, M., Notes, pp. 16-68.
Friedman, M., “The Marshallian Demand Curve,” Journal of Political Economy, December 1949, reprinted in Essays in Positive Economics.
Hicks, J. R., Value and Capital, Part I.
*Hicks, J. R., A Revision of Demand Theory.
*Slutsky, E., “On the Theory of the Budget of the Consumer,” Readings in Price Theory.
Knight, F. H., Risk, Uncertainty and Profit, ch. 3.
Schultz, H., The Meaning of Statistical Demand Curves, pp. 1-10.
Working, E. J., “What do Statistical ‘Demand Curves’ Show?”, reprinted in Readings in Price Theory.
Wold, H., Demand Analysis, ch. 1.
*Stigler, G.J., “The Early History of Empirical Studies of Consumer Behavior,” Journal of Political Economy, April 1954.
Friedman, M., Notes, pp. 69-75.
*Friedman, M., Notes, pp. 69-75.
*Friedman, M., and Savage, L. J., “The Utility Analysis of Choices Involving Risk,” reprinted in Readings in Price Theory.
Friedman, M., “The Expected Utility Hypothesis and the Measurability of Utility,” Journal of Political Economy, December 1952, pp. 463-474.
*Alchian, A., “The Meaning of Utility Measurement,” American Economic Review, March 1953, pp. 28-50.
Friedman, M., “Choice, Chance, and the Personal Distribution of Income,” Journal of Political Economy, August 1953, pp. 277-290.

 

III. SUPPLY OF PRODUCTS

Friedman, M., Notes, pp. 75-132.
Marshall, A., Book V, chs. 3, 4, 5, 12, Appendix H.
*Viner, J., “Cost Curves and Supply Curves,” reprinted in Readings in Price Theory.
*Robinson, J., Economics of Overhead Costs, ch. 9.
Robinson, J., “Rising Supply Price,” in Readings in Price Theory.
Apel, H., “Marginal Cost Constancy and Its Implications,” American Economic Review, December 1948, pp. 870-885.
*Coase, R. H., “The Nature of the Firm,” in Readings in Price Theory.
Chamberlin, E., The Theory of Monopolistic Competition, chs. 3, 4, 5.
Stigler, G. J., “Monopolistic Competition in Retrospect,” in Five Lectures on Economic Problems.
*Triffin, R., Monopolistic Competition and General Equilibrium Theory, esp. Part II.
Harberger, A. C., “Monopoly and Resource Allocation,” Proceedings of the American Economic Review (May 1954).
Stigler, G. J., “Competition in the United States,” in Five Lectures on Economic Problems.
Stigler, G. J., “The Statistics of Monopoly and Merger,” Journal of Political Economy, February 1956.
Stigler, G. J., “The Kinky Oligopoly Demand Curve and Rigid Prices,” in Readings in Price Theory.
*Robinson, E.A.G., Monopoly.
*Robinson, E.A.G., The Structure of Competitive Industry.
*Plant, A., “The Economic Theory Concerning Patents for Inventions,” Economica, February 1934.

 

Source: Hoover Institution Archives, Milton Friedman Papers, Box 77, Folder 1 “University of Chicago Econ 300A & B”.

Image Source: Photo credited to Joe Sterbenc/Becker Friedman Institute published on-line with the story “University mourns Gary Becker,” The Chicago Maroon May 6, 2014.

 

Categories
Exam Questions Harvard

Harvard. A.B. Honors Degree Examination in Mathematical Economic Theory, 1939

Today’s posting is a transcription of the “correlation examination” questions for mathematical economic theory given at Harvard in May 1939.

A printed copy of questions for twelve A.B. examinations in economics at Harvard for the academic year 1938-39 can be found in the Lloyd A. Metzler papers at Duke’s Economists’ Papers Project. 

Concentrators in Economics will have to pass in the spring their Junior year a general examination on the department of Economics, and in the spring of their Senior year an examination correlating Economics with either History or Government (this correlating exam may be abolished by 1942), and a third one on the student’s special field, which is chosen from a list of eleven, including economic theory, economic history, money and banking, industry, public utilities, public finance, labor problems, international economics, policies and agriculture.
Courses in allied fields, including Philosophy, Mathematics, History, Government, and Sociology, are suggested by the department for each of the special fields. In addition, Geography 1 is recommended in connection with international policies or agriculture.
[SourceHarvard Crimson, May 31, 1938]

Economic Theory,
Economic History Since 1750
Money and Finance,
Market Organization and Control,
Labor Economics and Social Reform.

  • One of the Six Correlation Examinations given to Honors Candidates. (May 12, 1939; 3 hours)

Economic History of Western Europe since 1750,
American Economic History,
History of Political and Economic Thought,
Public Administration and Finance,
Government Regulation of Industry,
Mathematical Economic Theory.

_____________________________________

If you find this posting interesting, here is the complete list of “artifacts” from the history of economics I have assembled. You can subscribe to Economics in the Rear-View Mirror below. There is also an opportunity for comment following each posting….

_____________________________________

 

DIVISION OF HISTORY, GOVERNMENT, AND ECONOMICS
CORRELATION EXAMINATION

Mathematical Economic Theory

(Three hours)

 

Answer either FOUR or FIVE questions, but not more than THREE from either group. If you answer only FOUR questions, write about one hour on ONE of the questions in Group B and mark your answer “Essay.” This question will be given double weight.

A

  1. A consumer’s indifference map for two goods X and Y is defined by \large\frac{4-\sqrt{y+1}}{x+4}=a
    Draw a graph showing the five indifference curves for the values 2, 3, 4, 5, 6 of the parameter a. Verify that they are of “normal” form.
  2. A business produces an income of $x this year and $y next year, where these values can be varied according to the relation y=1000-\frac{{{x}^{2}}}{250} . Show how \left\{ \left( -\frac{dy}{dx} \right)-1 \right\} can be interpreted as the marginal rate of return over cost. Show that the value of this marginal rate is \frac{x-125}{125} when this year’s income is $x.
  3. The market demand for a good is given by p=\beta -\alpha x . The market is supplied by two duopolists with cost functions {{\pi }_{1}}={{a}_{1}}x_{1}^{2}+{{b}_{1}}{{x}_{1}}+{{c}_{1}} and {{\pi }_{2}}={{a}_{2}}x_{2}^{2}+{{b}_{2}}{{x}_{2}}+{{c}_{2}} . Assuming that the “conjectural variations” are zero, show that the reaction curves are straight lines. Deduce the equilibrium output of each duopolist.
  4. (a) A steel plan is capable of producing x tons per day of a low grade steel and y tons per day of a high grade steel, where y=\frac{40-5s}{10-x} . If the fixed market price of low grade steel is half of that of high grade steel, show that about 5½ tons of low grade steel are produced per day for maximum total revenue.
    (b) The steel producer described in Section (a) monopolizes the sale of both quality steels. If the prices of low and high grade steel are $px and $py per ton, the demands are {{p}_{x}}=20-x and {{p}_{y}}=25-2y . Find an equation giving the output x of low grade steel for maximum total revenue. Show, by a graphical method, that just under 6 tons of this steel are produced per day.
  5. Given the marginal utility equations
    {{x}_{1}}d{{y}_{1}}+{{y}_{1}}d{{x}_{1}}\ge 0 , {{x}_{2}}d{{y}_{2}}+{{y}_{2}}d{{x}_{2}}\ge 0 ,
    for which the indifference curves are the rectangular hyperbolas {{x}_{1}}{{y}_{1}}={{c}_{1}} and {{x}_{2}}{{y}_{2}}={{c}_{2}} , given initial amounts a1, b1, and a2, b2 for the individuals (1) and (2) respectively, show that the equilibrium point for a single price transaction is
    {{x}_{1}}=\frac{{{a}_{1}}+p{{b}_{1}}}{2p} , {{y}_{1}}=\frac{{{a}_{1}}+p{{b}_{1}}}{2p} , where p=\left( \frac{{{b}_{1}}+{{b}_{2}}}{{{a}_{1}}+{{a}_{2}}} \right) .
  6. The demand for tea is {{x}_{1}}=40\frac{{{p}_{2}}}{{{p}_{1}}} and for coffee {{x}_{2}}=10\frac{{{p}_{1}}}{{{p}_{2}}} thousand lbs. per week, where p1 and p2 are the respective prices of tea and coffee in pence per lb. At what relative prices of tea and coffee are the demands equal? Draw a graph to show the shifts of the demand curve for tea when the price of coffee increases from 2s to 2s 6d and to 3s per lb.
  7. The production function is x=A{{a}^{\alpha }}{{b}^{\beta }} , where A, \alpha and \beta are constants. If the factors are increased in proportion, show that the product increases in greater or less proportion according as \left( \alpha +\beta \right)  is greater or less than unity. How is this property shown on a vertical section of the production surface through 0 and a given point on the surface? What is the special property of the case \alpha =1-\beta ?
  8. Examine the utility function u=\frac{x+a}{c-\sqrt{y+b}} , where a, b and c are positive constants, and show that the indifference map is a set of parabolic arcs and of normal form for certain ranges of values of the purchases x and y.
  9. If a{{x}^{2}}+b{{y}^{2}} = constant is the transformation function for two goods X and Y, that the marginal rate of substitution of Y production for X production is \frac{ax}{by} and that the elasticity of substitution is always unity.
  10. A monopolist produces cheap razors and blades at a constant average cost of 2s per razor and 1s per dozen blades. The demand of the market per week is {{x}_{1}}=\frac{10}{{{p}_{1}}{{p}_{2}}} thousand razors and {{x}_{2}}=\frac{20}{{{p}_{1}}{{p}_{2}}} thousand dozen blades when the prices are p1 (shillings per razor) and p2 (shillings per dozen blades). Show that the monopoly prices, fixed jointly, are 4s. per razor and 2s. per dozen blades.

 

B

  1. (a) A radio manufacturer produces x sets per week at a total cost of \$\left( \tfrac{1}{25}{{x}^{2}}+3x+100 \right) . He is a monopolist and the demand of his market is x=75-3p , when the price is $p per set. Show that the maximum net revenue is obtained when about 30 sets are produced per week. What is the monopoly price? Illustrate by drawing an accurate graph.
    (b) In the case of Section (a), a tax of $k per set is imposed by the government. The manufacturer adds the tax to his cost and determines the monopoly output and price under the new conditions. Show that the price increases by rather less than half the tax. Find the decrease in output and monopoly revenue in terms of k.
    Express the receipts from the tax in terms of k and determine the tax for maximum return. Show that the monopoly price increases by about 33 per cent. when this particular tax is imposed.
  2. (a) If u={{x}^{\alpha }}{{y}^{\beta }} is an individual’s utility function for two goods, show that his demands for the goods are x=\frac{\alpha }{\alpha +\beta }\frac{\mu }{{{p}_{x}}} and y=\frac{\beta }{\alpha +\beta }\frac{\mu }{{{p}_{y}}} where px and py are the fixed prices and \mu the individual’s fixed income. Deduct that the elasticity of demand for either good with respect to income or to its price is equal to unity.
    (b) The incomes of an individual in two years are {{x}_{0}} and and {{y}_{0}} his utility function for incomes is u={{x}^{\alpha }}{{y}^{\beta }} . Show that the demand \left( x-{{x}_{0}} \right) for loans this year decreases as the given market rate of interest 100r per cent. increases. Deduce that the individual will not borrow this year at any (positive) rate of interest if {{y}_{0}}<\frac{\beta }{\alpha }{{x}_{0}} .
  3. Discuss this quotation from Chamberlin: “We must conclude that the problems of proportion [among the factors] and of size cannot ordinarily be separated. The goal of the entrepreneur is not to discover the most efficient proportions and then to reproduce these continuously until the most efficient size is secured.”
  4. Discuss this quotation from Douglas: “Since the demand curves for labor and capital tend to approximate and to conform to the respective marginal productivity curves, it follows that an increase of 1 per cent in the quantity of labor would, other things being equal, normally tend to be followed by a decrease of ¼ per cent in the rate of wages.”
    [He uses a production function P=1.01\,\,{{L}^{{\scriptstyle{}^{3}\!\!\diagup\!\!{}_{4}\;}}}{{C}^{{\scriptstyle{}^{1}\!\!\diagup\!\!{}_{4}\;}}} ]
  5. Discuss this quotation from Broster: “Secondly, the chief concern of the railways is the maximization not of gross but of net revenue the maximum values of which are not the simultaneous product of the same level of fares. As is well known, the fare that attracts the former is that which corresponds to unit elasticity. It is perhaps not so well known—especially amongst the managers of sales departments—that except where the total cost remains constant for different rates of output of services, the fare that attracts the maximum net revenue is necessarily higher.”
  6. Discuss this quotation from Hicks: “The elasticity of substitution of labor for capital is the same as the elasticity of substitution of capital for labor.”

 

Source: Duke University. David M. Rubenstein Rare Book & Manuscript Library. Lloyd Appleton Metzler Papers. Box 7. [Harvard University], Division of History, Government and Economics. Division Examinations for the Degree of A.B., 1938-39.

 

Categories
Columbia Economists

Columbia. Richmond Mayo-Smith. Life and Death, 1854-1901

Material from Richmond Mayo-Smith’s course at Columbia, Historical and Practical Political Economy (1891-92),  was posted earlier. Below some biographical information from his entry to a four volume collection of portraits and biographical sketches of distinguished university graduates published between 1898-1900 which is followed by the report of his death and funeral ceremony in the Columbia University newspaper, The Columbia Spectator.

The circumstances certainly point to suicide. According to the Encyclopedia of World Biography (2004), “Following a crippling boating accident, Mayo-Smith sustained a nervous breakdown and committed suicide a few months later in New York City.”

Here a link to his colleague E.R. Seligman’s 1919 tribute to Mayo-Smith published in Vol XVII of Memoirs of the National Academy of Sciences (1924).

A few pages in the paper by David John Gow [“Quantification and Statistics in the Early Years of American Political Science, 1880-1922”, Political Methodology, Vol. 11, No. 1/2 (1985), pp. 1-18] help to put Richmond Mayo-Smith within a larger context.

Incidentally Mayo-Smith’s wife, Mabel Ford, was the daughter of Gordon Lester Ford, editor of The New York Tribune, and the great granddaughter of Noah Webster according to her New York Times obituary of 4 February, 1938.

The pioneer of applied demand analysis, Henry L. Moore, was hired in 1902 to fill Mayo-Smith’s position. “Genealogically” speaking, we could think of this year’s economics Nobel laureate, Angus Deaton, as a direct descendent in the line: Mayo-Smith to Henry L. Moore to Henry Schultz to Milton Friedman (to name only one of the numerous legitimate heirs of Henry Schultz) down to Deaton, so Richmond Mayo-Smith was Angus Deaton’s great-great grandfather as far as applied consumption DNA can tell.

_____________________________

MAYO-SMITH, Richmond, 1854-

Born in Troy, O., 1854; received his early education in the public schools and High School of Dayton; A.B., Amherst, 1875; studied in Berlin, 1875-77; and at Heidelberg during the summer term of 1878; Assistant in Political Science at Columbia, 1877-78; Adjunct Professor History and Political Science, 1878-83; Professor of Political Economy and Social Science since 1883.

RICHMOND MAYO-SMITH, M.A., Professor of Political Economy and Social Science at Columbia, was born in Troy, Ohio, February 9, 1854. Through his father Preserved Smith, he is descended from the Rev. Henry Smith, who came to this country during 1638 and took up ministerial work at Wethersfield, Connecticut. His mother was Lucy Mayo. He received his early education in the public schools of Dayton, Ohio and at the Dayton High School, entering Amherst College in 1871 and graduating in 1875. He studied abroad at the University of Berlin during the two years following, and also at Heidelberg during the summer term of 1878. He was appointed Assistant in Political Science at Columbia in 1877, and was promoted to Adjunct Professor History and Political Science in the following year. In 1883 he was elected to his present position in the Chair of Political Economy and Social Science. Professor Mayo-Smith married, June 4, 1884, Mabel Ford. They have four children: Lucy, Amabel, Richmond and Worthington Mayo-Smith. He is a member of the Century, University and Authors’ Clubs, and is not actively interested in politics.

Source: University and their Sons. History, Influence and Characteristics of American Universities with Biographical Sketches and Portraits of Alumni and Recipients of Honorary Degrees. Editor-in-chief, General Joshua L. Chamberlain, LL.D.  Boston: R. Herdon Company.  Vol. 2, 1899, pp. 582-3.

_____________________________

 

PROF. MAYO-SMITH

Death by Fall from Fourth Story Window of His Home
Funeral Yesterday Morning
An Appreciation by Professor Giddings.

The sad story of the sudden death of Richmond Mayo-Smith, Ph. D., Professor of Political Economy and Social Science is by this time well known. For several months he had been ill with nervous prostration and was taking his seventh year of rest from university work. At six o’clock Monday evening his wife and daughter left him resting in his study on the fourth story of his home at 305 West Seventy-seventh street. Fifteen minutes later he was found dead on the flagging in the rear of the house. It is supposed that in opening the window to air the room, which was very warm, he slipped on the hard wood floor and fell out.

Richmond Mayo-Smith had been a professor of political economy at Columbia since 1883. He was born in Ohio, and was graduated from Amherst College in 1875. After leaving Amherst College he studied for two years in Berlin University. While abroad he also was a tutor in Heidelberg University. His connection with Columbia began in 1877, when he was called to the University as a teacher of history. The year following he was made an adjunct professor, and in 1883, as stated, he was made a Professor of Political Economy and Social Science. He was an honorary fellow of the Royal Statistical Society of Great Britain and a member of the National Academy of Sciences. He was a writer on economic subjects and the author of “Emigration and Immigration,” “Sociology and Statistics,” “Statistics and Economics.” These works were published in 1890, 1895, and 1899.

Professor Mayo-Smith was a member of the Century, University, Authors’, and Barnard Clubs, and of the Amherst College Association. He was a vestryman i’ Christ Episcopal Church, Seventy-first street and Broadway, of which the Rev. Dr. Jacob S. Shipman is pastor.

 

On Tuesday Acting-President Butler issued the following order :

President’s Room, Nov. 12, 1901.

As a mark of respect to the memory of Professor Richmond Mayo-Smith, for twenty-four years an officer of Columbia University, and in token of the affectionate regard in which he was held by his colleagues and by the student-body, the exercises of the School of Political Science will be suspended until Friday morning, Nov15.

On Thursday, Nov. 14, the day appointed for the funeral of Professor Mayo-Smith, the exercises of Columbia College and of the Schools of Law and Philosophy, will be suspended entirely; the exercises of the Schools of Applied Science and Pure Science will be suspended until 1:30 o’clock P. M.

The Trustees of Columbia College, members of the University Council, and all officers and students of the University, are invited to attend the funeral services of Professor Mayo-Smith in a body, on Thursday morning, Nov. 14, at 10 o’clock A. M., in Christ Church, Broadway and 71st St. Officers and students of the University will assemble in the basement of the Church, entrance on 71st St., at 9:45 A. M. Professor James Chidester Egbert. Jr., is designated to act as Marshal.

Nicholas Murray Butler,

Acting President.

 

The funeral services were held yesterday morning in Christ Church and were conducted by Chaplain Van De Water and Dr. Shipman. President Low and Acting-President Butler followed the coffin, and members of the faculty and student-body, about 200 in number, followed in procession.

 

Professor Franklin H. Giddings has written the following tribute:

The death of Professor Richmond Mayo-Smith has made a gap in Columbia’s Faculty which no mere closing-up of the ranks can ever fill or conceal. Some losses are irretrievable. The place that a great man has held may be taken by another. In a nominal sense his work may be done by another; but it is never the same work. Professor Mayo-Smith was a man in whom rare gifts were in a very rare way combined. No one came under his personal influence, or into the circle of his friends, who did not recognize the accuracy of his knowledge, and the remarkable poise of his judgment; who did not soon feel the singular beauty and kindliness of his nature.

At Columbia Professor Mayo-Smith had taught history, political economy and statistics, and he had long served in the University Council. As a teacher he presented every subject with the utmost clearness. He insisted upon accuracy and thoughtfulness in all required work; but his judgment of students was marked by great considerateness and fair-mindedness. Students taking advanced work under his direction were admitted to his friendship and confidence, and he never ceased to take a deep personal interest in their success.

As a scientific investigator Professor Mayo-Smith made important contributions to both political economy and social science. His most distinguished work was in the domain of statistics, and there he stood easily first among Americans, and was recognized by Europeans as ranking with the three or four greatest names on the Continent and in England. The characteristics of his scientific work, as of his teaching, were scrupulous accuracy, perfect clearness of presentation, and that balanced judgment which is the highest mark of the scientific mind. He never attempted to make figures prove anything. With endless patience he sought to read in them their own sincere story, to discover so much of truth as he might; content always to admit that neither he nor any one else knew half the things that scientific investigators are commonly supposed to have discovered.

To all these interests of the teacher and the scholar Professor Mayo-Smith added the activities of the citizen, which he discharged in a way that was an inspiring revelation to all who knew him of his deep sense of duty. For many years a most valued member of the Central Council of the Charity Organization Society, he was also chairman of the Eighth District Committee, and he made it his business to know the exact facts about every case that came before the committee for consideration or relief. I have personally known of instance after instance in which his feeling of obligation to the suffering was discharged only by an expenditure of time and energy in visitation which I felt sure he could ill afford to give. In social life he was one of the most charming of men, whose delightful humor made him always in demand as the toastmaster or chief speaker whenever, in meetings of the many learned societies to which he belonged, relaxation and good-fellowship succeeded the more serious business of the occasion. As a friend he was unselfish to a degree, thinking always first of others, always last of himself. My own obligation to him is one which no words of tribute can ever repay.

Source:  Columbia Spectator, Vol. XLV., No. 1 (Friday, November 15, 1901), p. 1.

 

Categories
Columbia Exam Questions

Columbia. Exam Questions for Economics Ph.D. Candidates, 1949

After reading this examination that provides prospective candidates ample opportunity to tailor their examinations to their own course work and interests, one cannot help but conclude that the examination was not intended to test mastery of a common core nor a familiarity with a wide-range of economics. Maybe the explanation is that this is merely the reflection of a committee of individuals too polite to insist on cutting questions outside of one’s own specialization yet too stubborn to allow cuts either.

Perhaps you would like to leave in the comments the list of five questions you would choose to answer (assuming that you are eager to maximize your grade on this exam). 

I’ll go first: (5, 8, 16, 22, 26).

Your turn! (Scroll down the page to add comment)

__________________________________

EXAMINATION
for
PROSPECTIVE CANDIDATES FOR THE DEGREE OF PH.D. IN ECONOMICS

(May 7, 1949, 2:00 p.m. to 5:00 p.m.)

Questions on Specific Areas of Economic Study

Answer any FIVE but NOT MORE THAN FIVE questions.

A. Write all answers legibly in ink or on a typewriter.
B. Begin each question on a fresh sheet of paper. Write your name on all sheets used.
C. Be as specific as the question permits.
D. Be sure that your statements are relevant to the question.
E. Allow yourself time to reread your answers before handing in the sheets.

__________________________________

  1. What part do psychological and technological factors play in the propagation of business cycles?
  2. Is there a need for a separate theory of international trade, and if so, on what grounds?
  3. Some writers attribute the dollar shortage to the greater increase of productivity in the United States as compared to the rest of the world (including, particularly Western Europe). What do you think of this explanation?
  4. Discuss the origins of money (a) as means of payment, and (b) as means of exchange.
  5. Describe the present distribution of powers, among the various agencies concerned, over the American currency and commercial banking systems. In particular, who determines the volume of “money”?
  6. Describe and appraise both the theoretical and the historical arguments for the “stagnation” thesis.
  7. Discuss the main trends in Soviet foreign trade under the pre-war five-year plans.
  8. To what extent is the predominance of agriculture in the productive activities of a country correlated with a low level of income? Explain the relation between the level of income and the importance of agriculture.
  9. Explain the principal influences affecting the amount of capital in different countries.
  10. What was the role of fiscal policy in mercantilism?
  11. Give a critique of the role of marginalism in John R. Commons’ institutional economics.
  12. Discuss the place of the Most-Favored-Nation Clause in the French Commercial Policy in connection with the tariff legislation of 1892 and of 1919.
  13. Give an account of the organization of labor on the Soviet collective farm.
  14. Comment on the methods and conclusions of Seligman’s “Shifting and Incidence” in the light of recent developments in the field of economic analysis.
  15. Discuss the probably future role of “benefit taxation” in the light of prevailing trends of fiscal policy.
  16. Suppose that list prices of new automobiles had been raised by 15% above actual levels prevailing in 1946. What would have been the effect on used car prices?
  17. “There is a disposition on the part of the judiciary to emphasize the compensation of the injured person rather than the punishment of the guilty one.” Outline three major features of the workmen’s compensation laws that justify this generalization. Discuss one important exception.
  18. “The expanding commercial interests of the merchant capitalists of the New England colonies came into conflict with the great capitalistic interest of British West Indian sugar.”….”On the contrary, these two interests were not competitive but essentially complementary.” Write a careful statement of the problem raised by these two conflicting assertions.
  19. Assume that you were a capitalist in 1830 with money to invest in manufacturing either in Great Britain or in the United States. Indicate the considerations (other than patriotism) that you would have weighed in choosing between the two countries for your investment.
  20. What is the “parity” concept as it applies to agricultural prices? Discuss the implications, for economic processes at large, of the application of price parity to agricultural commodities, and give your appraisal of the parity program.
  21. Discuss the changes that have occurred in the United States since the First World War, in the volume and character of production and in productivity. What effect, if any, may these changes be expected to have on the operating characteristics of the American economy and on the problems involved in maintaining full employment?
  22. Explain how statistical evidence may be used in applying tests of significance in economic research. What is the logical justification of such tests? What constitutes “verification” or “proof” in scientific inquiry?
  23. In public utility rate making, how my one distinguish between differences in rates for various types of service that are “discriminatory,” and differences that are “non-discriminatory”?
  24. Distinguish between the capitalization (security structure) of (a) a typical manufacturing corporation, (b) a typical electrical utility company, and (c) a typical railroad company.
  25. Discuss the merits of the contention that, in the determination of the net income of a business enterprise, the allowance for depreciation should be based on the replacement cost of the depreciating asset rather than on original cost.
  26. “Wages are governed by the marginal productivity of labor.” Explain as specifically as possible what this means, in operational terms. Do you consider the statement valid?
  27. Assuming an industry consisting of a limited number of large producers selling a standardized product. Is it possible to construct a theory which will define the level to which price will naturally tend? Indicate one or more answers which have been given to this problem, and problems involved with respect to assumptions used.
  28. In what practical ways did the Wagner Act contribute to the growth of American unions since 1935?
  29. In what respect and why does the Taft Harley Act change the status of foremen in American labor law?

 

Source: Columbia University Library, Manuscript Collections. Albert Gailord Hart Collection. Box 62: “Teaching Materials 4. Columbia University.” Folder: Label “TEACHING Sec 4: Col Univ Ec 103/4 MICRO: EXAMS”.

Image Source: Alma mater before the old library of Columbia University from Historic Preservation Education Foundation.

Categories
Economists Harvard

Harvard. Charles F. Dunbar Obituary by Taussig, 1900

In this posting I first provide links to six successive editions of Charles F. Dunbar’s textbook on the theory and history of banking (four of which revised and expanded posthumously). 

Next, following President Eliot’s memoir from the last posting, I append here the obituary for Charles F. Dunbar written by Frank W. Taussig published February 3, 1900 in the Cambridge Tribune.  

Another memoir (written by Edward H. Hall) regarding Dunbar was published in vol. 14 (1900-01) of the Proceedings of the Massachusetts Historical Society, pp. 218-228.

__________________________________

Dunbar, Charles F. Chapters on Banking, privately printed in Cambridge, Massachusetts.

Zero edition, 1885. Eight chapters printed for the use of classes in Harvard University.

Dunbar, Charles F. Chapters on the Theory and History of Banking. New York, G. P. Putnam’s Sons, The Knickerbocker Press.

First edition, 1891. Added introductory chapter, chapter on combined reserves or the system of Clearing-House loan certificates and one on the Bank of Amsterdam.

Note: title shortened for second through fourth edition to The Theory and History of Banking.

Second edition, 1901. Enlarged and edited by O. M. W. Sprague.

Third edition, 1917. Enlarged by Oliver M. W. Sprague to include three new chapters on Foreign Exchange, Central Banks and on the Federal Reserve System.

Fourth edition, 1922. With chapters on foreign exchange and central banks by Oliver M. W. Sprague and a supplementary chapter presenting the record of the Federal Reserve System by Henry Parker Willis.

Fifth edition, 1929 With supplementary chapter presenting the record of the Federal Reserve System by Henry Parker Willis. Revised and in part rewritten with additional material by Oliver M. W. Sprague. New York: G.P. Putnam’s Sons.

__________________________________

Charles Franklin Dunbar.

Charles Franklin Dunbar, professor of Political Economy in Harvard University, died at his house, on Highland street, on the night of Monday, January 29. He had been ill for some weeks, but his friends had not thought the end would come so soon.

Professor Dunbar was born July 28, 1830, and graduated from Harvard College in 1851. Among his class-mates were Professors Goodwin and Langdell, and Dr. S. A. Green; and, among those whom death has already carried away, Professor W. F. Allen of the University of Wisconsin, General Francis W. Palfrey, and Messrs. George O. Shattuck and Augustus T. Perkins of Boston. Professor Dunbar studied law after graduation, but in a few years became connected with the Boston Advertiser. To that paper he gave some of the best years of his life. He became editor-in-chief in 1862, and so was in charge of the paper during the greater part of the civil war. While always independent in his judgments, he was a fervid supporter of the Union cause, and many of his editorials rang through New England like a trumpet blast. In 1869 the paper changed hands, and Professor Dunbar resigned as editor and disposed of his interest. He was invited shortly to accept the professorship in Harvard University, and after two years spent in travel and study in Europe, assumed the duties of the professorship, in 1871. For the rest of his life he was in active service, and a resident of Cambridge through these thirty years.

Professor Dunbar’s sagacity and tact led to his selection for important administrative offices. He was dean of the Faculty of Harvard College from 1876 to 1882, and was the first dean of the Faculty of Arts and Sciences in Harvard University when that body was organized in 1890. He remained its dean till 1895. From 1886 to 1896 he was editor of the Quarterly Journal of Economics, established by the University in the first named year.

Professor Dunbar was distinguished by solidity of learning, sanity of judgment, independence of views, and scholarly thoroughness in probing to the bottom every subject he took up. His favorite topics were banking, currency and financial administration. He was interested alike in the history of these subjects and in current problems connected with them. Probably no man was better equipped by attainments and by justness of views to give advice on the financial questions which have been before the American community for the last thirty years. His writings on them gave but fragmentary indication of his attainments. He published a compact volume on the “Theory and Practice of Banking,” [sic, correct title is “The Theory and History of Banking”] which, though brief, is the best book on this subject in the English language, and in some respects perhaps the best in any language. To the Quarterly Journal of Economics he contributed frequent articles on financial subjects, and on some questions of theory; and he had abundance of material for others which he had planned but unhappily was not able to prepare. His administrative duties absorbed a large share of his strength, and stood in the way of the execution of his literary plans.

Professor Dunbar was a member of the Massachusetts Historical society, and a Fellow of the American Academy of Arts and Sciences. He was at one time president of the American Economic association. His reputation abroad was at least as high as it was in the United States: among economists the world over he was known as a scholar of the first rank.

An easy and graceful writer, he was also a clear and attractive lecturer, with a remarkable faculty for the consecutive and systematic exposition of difficult subjects. The weakness of his voice was the only obstacle which ever stood in the way of the interest and easy comprehension of his lectures. With small classes of advanced students he was at his best, and no one who came in contact with him under such circumstances failed to cherish feelings of admiration and affection far him. Staunch in his own opinions, open-minded as to those of others, free from all vanity or ostentation, strong in his affection for those with whom he came into close association, he left a memory which will be cherished by his associates and former students.  F. W. Taussig.

Source: Cambridge Tribune, Vol. XXII, No. 48 (February 3, 1900), p. 4.

Image Source: From cover of the 1885 copy privately published Chapters of Banking.

Categories
Economists Harvard

Harvard. Biographical memoir of Charles F. Dunbar, 1900

Only a few days after Harvard’s first professor of Political Economy died on January 29, 1900, the President of Harvard himself, Charles William Eliot, read the following memoir of the life of Charles Franklin Dunbar before the Massachusetts Historical Society on February 8, 1900. The memoir was published in The Harvard Graduates’ Magazine‘s June, 1900 issue and reprinted along with biographical sketches of other “Sons of the Puritans” in 1908. Page numbers for the 1908 reprint of the memoir have been placed within square brackets below.  The account of Dunbar’s Harvard career begins on page 75.

_______________________

CHARLES FRANKLIN DUNBAR

By Charles W. Eliot

[59] Charles Franklin Dunbar, born at Abington in July, 1830, was of Scotch descent, as his sandy hair and complexion, his shrewdness, reticence, and quiet humor plainly testified. He was much interested in his family descent, and gave no little time to tracing it both in Scotland and in Massachusetts. In one of his journeys to Scotland he visited the chief seats of the Dunbar Clan in Morayshire, and found reason to believe that from and after the year 1400 Dunbar was one of the prevailing names in that region. The first Dunbar in Massachusetts was Robert Dunbar of Hingham, who said of himself, in a deposition he made in court in 1659, that he was a servant of Mr. Joshua Foote when Mr. Foote lived in Boston. By a series of careful investigations Charles Franklin Dunbar established the strong probability that this Robert Dunbar who was held to the services of Joshua Foote for a term of years as early as 1655, and possibly as early as 1652, was one of Cromwell’s Scottish prisoners taken at the battle of Dunbar in 1650, [60] or at the battle of Worcester in 1651. It is certain that some of the prisoners taken at the battle of Dunbar were sent to the Colony of Massachusetts Bay in 1650-51, after having endured frightful sufferings which killed three quarters of the prisoners originally captured. Robert Dunbar, who died in Hingham in 1693 at about sixty years of age, was therefore, in all probability, of very tough fibre.

The father of Charles Franklin Dunbar was Asaph Dunbar, who was born in 1779 and died in 1867. Charles was Asaph’s youngest child. He had three brothers, all of whom filled out a reasonable span of life, and two sisters, one of whom died in infancy and the other at the age of twenty-one. The father’s business was making boots and shoes, and Charles’s three older brothers grew up in that business in Plymouth County, but while still young went away to New Orleans to sell there the goods which their father manufactured. One of these three brothers returned to New York to establish himself there in the same business. Charles was the only one of the brothers who received a liberal education. He was sent to Phillips Academy, Exeter, — probably because he had always shown a strong desire to read and an aptitude for study. The [61] success with which he accomplished the academic course at Exeter determined his being sent to Harvard College, where he graduated with credit in 1851. The fact that he was sent to Exeter at thirteen years of age determined his subsequent career; and he always felt unbounded gratitude to that ancient academy, a gratitude which he expressed by serving it for many years as a member of the board of trustees. At Harvard College he won the respect and friendship of scores of young men, many of whom have come to the front in one way or another during the forty-eight years which have elapsed since he graduated. Some of them were associated with him in after life; and he always retained their warm regard and admiration.

After leaving college he went for a time to his brothers in New Orleans; but soon came back, first to New York and then to Boston, applying himself steadily to business. A threatening of serious trouble in the lungs obliged him to abandon this indoor occupation; whereupon he bought a farm at Lexington, and entered cheerfully on the quiet out-of-door life of a farmer, for which he developed a strong taste and aptitude. Here he soon recovered his health and strength; so that he took up the study of the law at the [62] Harvard Law School, and in the office of Ebenezer Rockwood Hoar, and was in due course admitted to the Suffolk bar in 1858. Practice coming to the young lawyer but slowly, he had ample time to write for the Boston Daily Advertiser, and, finding this occupation congenial, he became within a little more than a year editor and part owner of that influential newspaper. In this enterprise he was supported and helped by the occasional labors of a group of young men whom he had known at Exeter and in College; but he himself gave his whole time and strength to the paper. He remained in the position of editor for ten years, — all through the Civil War, and through the early years of reconstruction and gradual pacification. During the Civil War he personally wrote every editorial article in any way related to the war which appeared in that newspaper. The Advertiser became by common consent the leading paper in Boston, and no newspaper since has exercised the same influence in this community. His position brought him into contact with a large proportion of the leading men of the time in eastern Massachusetts, — with merchants, manufacturers, politicians, soldiers, lawyers, and preachers. He wrote, of course, constantly on military [63] events and prospects; but the subjects he best liked to deal with were financial, economic, or political, — such as the war loans, tariffs, and banking acts, the suspension of specie payments, and the measures taken to collect a great internal revenue. The amount and the quality of the work he did in the ten years between 1859 and 1869 were remarkable, considering that he began this work at twenty-nine and ended it at thirty-nine years of age. At thirty years of age he was wielding an influence which would now seem almost impossible of attainment at that age.

A few citations from his editorials will suffice to give an idea of the elevation of their tone, and of their moderation, judicial quality, and prophetic insight.

As early as July 4, 1861, he thus defined the objects of the war for the Union, and the spirit of the Northern people: —

“We are fighting now, as eighty-five years ago, to defend a cause in which the grandest principles of government and the highest interests of man are involved. Our people now as then have thrown aside all remembrances of old divisions, and have united in an enterprise which they believe to be just and holy. Life, fortune, and sacred honor [64] are again pledged to the support of the patriotic declarations with which the second war for liberty has been undertaken; and again has Congress assembled, prepared to forego the ordinary topics of political strife, to forget as is believed all tests save the one question of fidelity to country, and to take counsel in singleness of heart for the one great object.”

Immediately after the heavy defeat of the Union troops at the first battle of Bull Run, he wrote, July 23, 1861: —

“We said at the outset that this reverse had temporarily defeated the scheme for advancing through Virginia. Let no man to-day whisper the thought of abating a jot of our vast undertaking. Taught by one reverse the nation will rise above its misfortune, and press on in its just and holy cause. The people who have poured out their blood and treasure so freely will be kindled to new efforts. … Our present misfortune will disclose to all the true secret of our weakness, and will teach all that the advance for which some have so long clamored is not to be accomplished in a single effort. With a full knowledge on all hands of the nature of our undertaking, and with such further preparation as must now be made for this grand enterprise, we can doubt its final success as little as we can doubt the justice of the cause in which [65] it is undertaken, or the wisdom of the Providence which rules all things for our good.”

He early foresaw the fate of slavery as an institution. Writing on the last night of the year 1861 a survey of the events of the year, he made this prophetic utterance a year before the Emancipation Proclamation was issued: —

“It leaves our own people with renewed courage, united beyond all hope in support of the government in a most trying case, and fully ahve to the importance of closing the war at once. It also leaves the majority with an unshaken resolution to confine the war to its proper objects, and to sustain the President in the firm and conservative course which he has pursued through the ten months in which he has held office. At the same time, the year has demonstrated to our whole people the great fact, that in the designs of Omnipotence the South has been led through its own folly to write the doom of slavery. Heavier and heavier are the blows which descend upon that institution, and more and more significant are the proofs that the South built upon a weak foundation, when, within this very year, it announced slavery as the cornerstone of its fabric, political and social.”

Near the close of the year 1862 Secretary Chase communicated to the Committee on Ways [66] and Means the draft of a bill to provide the necessary resources for the prosecution of the war. The second section authorized the Secretary of the Treasury to borrow nine hundred million dollars in any of the modes heretofore authorized for making loans. The bill also contained the details of the national bank scheme. Mr. Dunbar’s comments on this bill are in part as follows: —

“The most important feature of this bill, so far as regards the immediate emergencies of the country, is the second section, and this it seems to us has been well conceived. . . . Should this power be granted by Congress, we trust that the secretary will use it with liberal forethought. Armed with full powers, he will be able to feed the market with such securities as are most popular, at times when prices are favorable. Unrestricted by needless trammels, he can avail himself of the most favorable proposals which may be suggested from time to time by those who have money to loan, or who can present well-considered plans for meeting the wants of the Treasury with the least cost to the nation.”

Of that very important part of the bill which related to the establishment of the national bank system he speaks as follows, in his few words [67] showing that he had a clear vision of the wide scope and far-reaching consequences of the project:—

“It has been taken for granted that this measure will provoke a violent opposition, which, nevertheless, as yet has not manifested itself in any very definite shape. It is nowhere denied that the Secretary’s plan insures several very decided advantages; it looks rather to the establishment of a sound currency for the country upon a permanent basis than to any immediate results. If it be said that it will be time enough to legislate to this end when we have got out of the war and the financial difficulties incident thereto, it may be answered with at least equal force that the necessity of reform will then be less generally apparent. ‘Why don’t you mend your roof ?’ asked a traveler of a negro in whose leaky hut he had taken refuge during the shower. ‘Cause it rains’ was the answer. ‘But why don’t you mend it at some time when there is no rain ?’ ‘Cause then it don’t leak.’ This sort of logic will hardly justify Congress in refusing a careful attention to Mr. Chase’s plan, notwithstanding the statement paraded in advance, that ‘the majority of the Ways and Means are hostile to Mr. Chase’s scheme,’ and that ‘this sentiment of disapproval cannot possibly be changed.’”

[68] After the great victories at Gettysburg and Vicksburg, July 3-5, 1863, Mr. Dunbar wrote as follows on the 8th of July: —

“We speak of these events as of extreme political importance, because they have now for the first time fairly established the ascendency of the national power over the rebellion. Hitherto the struggle has been often a drawn game, and even in our moments of success has left the military strength of the rebels so formidable as to keep their hopes alive. The handwriting is now on the wall in characters which the rudest may read, warning the rebels that henceforth theirs is a hopeless cause, and that from this time their efforts must decline. We may now, at any rate, count upon the moral effect of defeat and loss of faith in their cause, and may hope for the appearance of those discontents and divisions to which despondency gives rise, and which precede the final ruin of a cause which, like the rebellion, has no root in sound principle.”

Looking back on this statement after an interval of thirty-seven years, we are struck with its absolute accuracy.

In his review of the year 1863, on the 31st of December, his comments on the Proclamation of Emancipation illustrate the perfect balance of his judgment: —

[69] “The most distinctly marked event in the conduct of the war for the year, however, is unquestionably the Proclamation of Emancipation issued on the 1st of January, 1863. Of this measure it can now be said, that it has equally disappointed its advocates and its opponents. It has failed to effect the dissolution of the rebel power which was so confidently predicted as certain to be its instantaneous effect, and has left the actual work of emancipation to be performed by the steady advance of military operations. On the other hand, it has failed to make that disastrous division among the loyal which was predicted by many of its opponents. The mass of the people have acquiesced in it as a military measure taken in good faith. But we must remark, they have done this the more readily since on independent grounds the policy of emancipation has gained favor in the popular mind rapidly during the year.”

Speaking of the extraordinary sales of 5-20 bonds in the summer and autumn of 1863, he writes as follows: —

“Throughout the country these bonds have been eagerly sought, with the noblest demonstrations of confidence and affection towards the government in defense of which the money is contributed. The success with which the government now deals with a debt of great magnitude has inspired the country [70] with faith in its ability to cope with the future, heavy as are the burdens promised by the Secretary of the Treasury.”

How far-seeing is the following paragraph, which occurs in the same review of the year 1863: —

“The feelings of the French Emperor towards the United States had long been suspected, but were first fully appreciated by our people when his designs in Mexico were fairly unmasked, and when he announced his deliberate design of erecting a throne in that country to be occupied by a prince nominated by himself. It was immediately perceived that France had created for herself upon this continent an interest adverse to that of the United States. The occupation of the Rio Grande by our forces, however, together with the established certainty that the Emperor will for the present find enough to do in dealing with the Mexican people, who do not accede to the fiction that Maximilian is their choice, has finally quieted all fears as to the course of France for the present.”

In his review of the year 1864, Mr. Dunbar wrote as follows: —

“Never has the struggle seemed so gigantic as in this year, never have the contending forces so [71] convulsed the continent with their efforts, or so appalled the spectators of a strife as terrible and unrelenting as the of the elements. Indeed, this is an elemental strife, which we have seen approaching its climax and crisis, — a strife which, in the words of a philosophic observer who was lately among us, is waged ‘not only between Aristocracy and Democracy, between Slavery and Social Justice, but between ferocious Barbarism and high Civilization.

“It is only when we view the contest in this light that it is possible to realize completely the futility of such efforts at pacification as that which has characterized this year, and which was defeated by the will of the people a few weeks ago. These raging elements are as far beyond the reach of all such attempts to quiet their agitation as is the tempest which purifies the physical atmosphere. The forces have long been gathering, they are in the full height of their sublime power, and are not to be stayed until the mission assigned to them by Providence is accomplished. A great political party thought otherwise, and sought by months of carefully studied effort to still the contention by premature peace; and it finds itself to-day shivered to atoms, and its candidates swept aside like chaff and forgotten. The judgment of the nation and its will have risen to the height of the occasion, and have settled irrevocably the devotion of this [72] people to their grand task to the very end. In its moral aspects, then, the result of the election has been the great event of the year and of the war.”

Mr. Dunbar was often called upon to express the strongest emotions of the people under circumstances of tremendous excitement. After listening all day to the rejoicings in the streets of Boston over the surrender at Appomattox, he wrote at night an editorial in which two out of the four paragraphs are as follows: —

“Four years ago this morning we were obliged to say in this place ‘we do not seek to pierce the gloom which now seems to overspread the future.’ Four years of that future as they have enrolled themselves have shown many another crisis, or agony more acute, but none of gloom so depressing as settled on us all in that week of uncertainty. This day is the anniversary of the humiliating correspondence between General Beauregard and Major Anderson, in which he demanded the surrender of Fort Sumter as a foregone necessity. To-morrow is the anniversary of the day on which he opened his fire. These four years have called upon the nation to show its steadfast endurance. They have called for that loyalty to institutions which does not seek to pierce the gloom of the future. They have bidden the nation stand firm on the eternal principles of its government, [73] and trust God to give it victory, when for victory the time had come. Through that gloom, or the flushes of hope which at one moment or another varied it, the nation has stood firm, and at last the end has come. . . .

“Such are the moral advantages of the victory. They make a nation so strong that war in its future is wholly unnecessary, — it seems hardly possible. This nation is just, — it can be as generous as it is just. It has no entangling foreign alliances, it need have no petty foreign jealousies. God has shown it His mercy in a thousand ways, and now that He blesses it with Peace, it has His promise that Peace shall lead in every other angel of his Kingdom.”

At the close of the year 1865 he wrote as follows, prophesying a period of discussion and evolution which has not yet ended: —

“The year, we may trust, is the last in the succession of years which by striking and exciting events compete for the leading place in our annals. The period of great deeds is perhaps over; we now have remaining questions of magnitude to be debated and settled, or to be suffered to work towards their own solution by process of time, and not concentrating their fierce interest into single great transactions, of which we have known so many since 1860. The question as to the future [74] of the freedmen is not to be settled by the turn of any crisis, but by many discussions, the long-continued operation of opinions, and the progress of immigration, of industry, and of ideas. Financial questions, of which we have so many of importance, are as little to be determined by any special action, but cast their shadow far over the coming years. The foreign questions, of which the closing year leaves us a supply not trifling in importance if scanty in number, are as little likely, we may hope, to assume such form as to bring back the unhealthy excitements which have long been familiar, but will rather relapse into the ordinary course of international litigation, or be settled by causes and influences which in power are far above the counsels of emperors. In short, we now enter in public matters upon a period of discussion; and if results appropriate to this method of action are wrought out with half the skill and power which we have seen displayed in the marvelous twelvemonth now ending, we shall find our prosperity and happiness, and our development in all that ennobles a people, settled on a foundation more solid than our fathers ventured to hope for.”

During his administration the Advertiser as a property increased greatly in value; so that when in 1869 Mr. Dunbar found it necessary again to pay attention to his health, and to give [75] up work for a time, he sold his interest in the newspaper for a sum which amounted to a competency for himself and his family. This was really a value which his own mental gifts and moral character had imparted to the newspaper. There is no more satisfactory way in which a man can earn a competent support for his family before he is forty years of age. All through his life Mr. Dunbar was a careful, frugal, and successful man of business, although he gave but a very small portion of his time to that side of life.

In order to recover from the nervous exhaustion which he experienced in 1868, he made two journeys to Europe, the first alone, but the second with his family. I had come into the Presidency of Harvard College in 1869, and one of the first measures which the Corporation resolved to prosecute with vigor was the establishment of a Professorship of Political Economy, and the selection of an incumbent for the chair. Mr. Dunbar being well known to all the members of the Corporation, the appointment was offered to him in 1869, and he gave a conditional acceptance to take effect two years later. A quiet life in various parts of Europe restored his health and gave him opportunity, [76] for the prosecution of studies which prepared him further for his new function; and In 1871 he took up the work of his professorship, to which he thereafter steadily devoted himself for more than twenty-eight years.

Professor Dunbar was the first Professor of Political Economy that Harvard University ever had. That great subject had previously been one of the numerous subjects assigned to the Alford Professor of Natural Religion, Moral Philosophy, and Civil Polity. Professor Dunbar announced for the year 1871-72 a course prescribed to Juniors on Rogers’s “Political Economy” and Alden’s “Constitution of the United States,” two hours a week for half a year, and an elective course in Political Economy for the Senior Class, based on Adam Smith’s “Wealth of Nations,” Bowen’s “American Political Economy,” and J. S. Mill’s “Political Economy;” but these courses were announced under the head of Philosophy. The elective course was attended by seventy-five Seniors. The next year his elective course appears under its proper heading, — Political Science, — the description of the course being altered to the following: J. S. Mill’s “Political Economy,” McCulloch on Taxation, Subjects [77] in Banking and Currency. Professor Dunbar also conducted in 1872-73 a required course for Juniors in Political Science, two hours a week during half a year. That year he used as textbooks for the Junior’s Fawcett’s “Political Economy” and the Constitution of the United States. In 1873-74 Professor Dunbar had for the first time the assistance of an instructor, because the required course in the Elements of Political Economy was transferred from the Junior to the Sophomore year, — on its way to extinction, — so that this required course had to be given that year to two large classes. Under Professor Dunbar’s elective course, Bagehot’s “Lombard Street “appears for the first time. In the next year Professor Dunbar gave, in addition to the prescribed Political Economy, two elective courses parallel to each other, one being preferable for students of History. The rapidly increasing number of students in the department made it desirable to offer these two parallel courses, so that neither class should be too large. One hundred and thirty-one students chose these electives. In 1875-76 Professor Dunbar was conducting three progressive courses: the prescribed elementary course, a first elective course on J. S. Mill’s “Political Economy,” [78] and the Financial Legislation of the United States; and an advanced course on Cairns’s “Leading Principles of Political Economy;” and McKean’s “Condensation of Carey’s Social Science;” and the number of students attending his course was steadily increasing. In the following year Professor Dunbar became Dean of the College Faculty, an administrative position which he held for six years. The prescribed course in Political Economy for Sophomores now disappeared. The elective courses were fully maintained. Professor Dunbar had some assistance in the elementary elective course, because of the necessity of devoting a good deal of his time to the administrative work of the Dean’s office. His assistant in the year 1877-78 was Mr. Macvane, now Professor of History in Harvard University. The next year his assistant was Dr. James Laurence Laughlin, who had the title of Instructor in Political Economy. In 1880-81 another course in Political Economy was added to the two already given, Professor Dunbar working in all three courses, but being assisted in the first two by Dr. Laughlin. The most advanced elective under Professor Dunbar was based on Cairns’s “Leading Principles of Political Economy,” [79] McLeod’s “Elements of Banking,” Bastiat’s “Harmonies Economiques.” In the year 1882-83 Professor Dunbar took leave of absence in Europe. His work was carried on by Dr. Laughlin and a new instructor, Mr. Frank W. Taussig, now Professor of Political Economy in Harvard University. A new half-course was added this year, — a course on the Economic Effects of Land Tenures in England, Ireland, France, Germany, and Russia. The next year brought considerable expansion to the Department. Professor Dunbar returned to his work; Dr. Laughlin was made an assistant professor; and Dr. Taussig offered for the first time a course on the History of Tariff Legislation in the United States. The number of courses offered by the Department suddenly expanded to four courses running through the whole year, and three running through half a year. Economic History appeared for the first time as part of the instruction given by the Department, Professor Dunbar having charge of the course. It was in that year that the plans of Professor Dunbar for the development of his department in the University became apparent to the academic world. Dr. Taussig soon became an assistant professor; Dr. Laughlin was [80] promoted to a full professorship at Cornell University, whence he was subsequently transferred to the University of Chicago; and a series of young men, all selected by Professor Dunbar, were brought forward in the Department as teachers. The number of teachers and courses increased until, in 1894-95, this Department, called Economics since 1892-93, employed three full professors, one assistant professor, and three instructors, and the number of courses had risen to six full courses and seven half-courses. In 1899 the lowest elective course in Economics was opened to Freshmen; so that the Harvard student thenceforth had access to that subject in all the four years of his college course. For the present year, 1899-1900, courses were announced which gave employment to three full professors, one assistant professor, and six instructors. In the academic year 1898—99 the choices made of courses in Economics numbered 1263.

Such was the development given in twenty-eight years to a subject which certainly should be second to none in value or dignity at an American university. At every step of the process it was Professor Dunbar’s sagacity, sobriety, and fairness which commanded confidence [81] and secured success. He thus made, in the course of twenty-eight years, as it were with his own hands, a complete collegiate instrument for training young Americans in Political Economy, the first such instrument ever constructed. If it should occur to any one that this growth was made possible by the general atmosphere at Harvard, the answer would be that Professor Dunbar had much to do with determining the quality of that atmosphere.

In 1886 a timely gift of a fund of $15,000 from one of Professor Dunbar’s pupils enabled the Corporation to establish the Quarterly Journal of Economics, published for Harvard University. They took this step by the advice of Professor Dunbar, and on the condition that he should edit the Journal. He acted as editor for ten years, and in that time established the position of the Journal in this country and in Europe as a valuable medium for economic discussions and researches. The subjects of some of the articles which he wrote for this Journal will indicate the wide range of his studies: In 1886, “The Reaction in Politics;” in 1887, “Deposits and Currency,” and a note on Ricardo’s Use of Facts; in 1888, a notice of an old tract entitled “The New-Fashioned Goldsmiths,” [82] a tract which appears to have been the source of the generally accepted statement as to the origin of private banking in London in the seventeenth century. In the same year appeared “Notes on Early Banking Schemes “from his pen, and an article on “Some Precedents Followed by Alexander Hamilton.” At the end of this last paper, after a learned review of the system advocated by Hamilton, and of the sources of the measures which he recommended, Professor Dunbar said in conclusion: “No statesman could have a greater task set for him, and political science can hardly have in store any greater triumph than this application of the experience of other men and other nations.” In 1889 he wrote for the Quarterly Journal an article on the Direct Tax of 1861, the conclusion of which was, “The direct tax provided for by the Constitution has at last been discredited as a source of revenue, and it has also been too prolific of misconception and confusion to have any Influence henceforth as a practical measure of finance.” A single sentence from an essay he published in the Journal in 1891 on the academic study of political economy admirably expresses the true conception of the function of an instructor in any moral [83] science: “That the student should learn to reason truly is of far more consequence than that he should perceive and accept any particular truth, and the real success of the instructor is found, not in bringing his students to think exactly as he does, — which is unlikely to happen, and, indeed, unnatural, — but in teaching them to use their own faculties accurately and with a measure of confidence.” In another passage in the same essay, speaking of the conditions under which an instructor may or may not be silent concerning his own beliefs, he says, “There are few men whose weight of authority is such as to compel any extraordinary caution in the declaration of their minds.” Those two statements are highly characteristic of Professor Dunbar’s habitual attitude towards his own students.

One may easily trace through all the activities of Professor Dunbar as a teacher and writer the effect on his mind of his ten years’ work as the editor of a daily paper during a period of startling and far-reaching military, financial, social, and political events; but it is interesting to observe that commercial and economic questions began to engage his attention some years before the war. Thus we find in the North American Review an article by him on the Danish [84] Sound Dues written as early as 1856, when he was twenty-six years of age. His services as a university teacher grew naturally out of the studies and interests of his early manhood.

Professor Dunbar was Dean of the old College Faculty for six years, from 1876 to 1882, and the first Dean of the new Faculty of Arts and Sciences from 1890 to 1895. He therefore gave a large amount of administrative service to the University. As an administrative officer he was prompt, efficient, and wise. One peculiarity he had which was rather trying to some of the many students and parents of students with whom he came into contact, — he was sometimes too reticent and silent. He would listen patiently to a long tale in which the narrator felt great interest, and take it all in, but hardly utter a word in reply. Sometimes, however, after his interlocutor had despaired of getting an answer, he would give a concise but comprehensive reply which showed how sympathetically he had apprehended the whole subject under discussion. Ordinarily patient and cautious, he was entirely capable of quick decision and prompt action. On a reconnaissance he was circumspect and thorough; but when he once made [85] up his mind how the land lay and how the adversary was intrenched, he moved on the position, in the safest possible way, to be sure, but with energy and persistence. As a rule, his aspect was serene and mild; but on occasion his face could become set, and from his blue-gray eyes there came a steel-like gleam dangerous to his opponent. In his judgment of others he was gentle, unless he became satisfied that some man he had been observing did not play fair, or was untrustworthy at the pinch; then he became stern and unrelenting. It was these qualities which made him the successful journalist that he was at thirty years of age. The Faculty was always afraid to take a step of which he did not approve, and seldom did so, unless his occasional infirmity of silence had concealed from them his opinion. They felt in him a remarkable sagacity combined with quick insight and unwavering disinterestedness; and they found him to be uniformly just. If he now and then betrayed a prejudice, they felt sure that he had good grounds for it, and were much disposed to share it with him. Every one who has seen much of the world will perceive how rare a combination of qualities was [86] embodied in this modest and retiring man, and will understand how great a loss the University has suffered in his death.

In addition to the solid satisfactions Mr. Dunbar derived from his forty years of professional work, he had great delights in his domestic life. He married, soon after leaving college, Julia Ruggles Copeland, of Roxbury, and he survived his wife only two months. Five children were born to them between 1855 and 1862, of whom three sons and a daughter survive their father and mother.

I have already mentioned the life of the young family at Lexington. When he became editor of the Advertiser, he moved, first, to Roxbury; but finding the inevitable exposures of returning to Roxbury from his office late at night (often after the omnibuses had ceased to run) too great for his strength, he moved to a small house on River Street, at the foot of Beacon Hill. This house was comparatively sunless, and, though close to Beacon Street, had no outlook whatever. It was a great Relight to him and his wife and his growing children to establish the household in 1872 in a spacious house on the hill which rises north of Brattle Street, Cambridge, not far from Elmwood, [87] a house which commanded a charming prospect, and was surrounded by fine trees. He had earned the luxury of fine prospects, abounding sun and air, and garden grounds, as product of the work of his own brain. His tastes and habits were simple, but refined. Luxuries and superfluities had no charm for him. He was fond of driving and sailing, but needed no elaborate equipment for obtaining these pleasures. He valued these sports mainly as means of getting into contact with the beauties of nature by land and by sea. He had the natural healthy enjoyment in food and drink, but always preferred simple things to elaborate, and was displeased by extravagance or excess.

In 1886 he bought the larger part of Bear Island, off Mount Desert, the smaller part being already occupied by the United States as the site of a lighthouse; and here he built in 1893 a cottage for the summer occupation of his family. When visiting friends on the neighboring shore of Mount Desert, he had often marked the beautiful form of this island, and admired the exquisite views it commanded in several directions. In deciding upon the site of his house on this island, it was his chief care to avoid impairing the aspect of the island from [88] the neighboring shores, — a thoughtful result which he perfectly achieved. All his life he had great pleasure in carpentering. He always had a carpenter’s bench in any house he occupied, and delighted in good tools and in using them with skill. He could build with his own hands fireplaces, corner buffets, desks, tables, and other pieces of furniture. At Bear Island he built a large boat-house with chambers in its upper story, doing most of the work with his own hands, after the heavy framing had been put up. He enjoyed thinning the woods which covered the northern shore of the island, and studying the flora and fauna of his isolated kingdom. A thrifty little spruce, looking as if it could easily resist all the ice and snow, all the gales, and all the droughts of that northern clime, a single graceful birch, a mountain ash loaded with red berries, or a clump of ferns, sufficed to give him great enjoyment. With reading and writing interspersed, such pleasures filled his summer days so completely and so happily that he seldom wished to leave his island. Friends came to stay with him; but he seldom cared to go far from his cottage, unless on a sail or a drive with one of his neighbors of the main island. There was no road on his island, [89] and hardly a path, except little tracks between the hummocks and ledges; and there were no sounds, except the beat of the waves on the rocky shores, the singing of birds, and the rushing of the wind through the trees. One of the peculiarities of the climate of the Maine coast had singular charm for Professor Dunbar. On almost every summer evening near sunset, there falls a great calm and stillness. No matter how boisterous the day may have been, near sundown there comes a widespread, profound silence, unspeakably grateful to such a temperament as his. The hills of Mount Desert, in full view from his island, reminded him of the similar hills built of primary rocks which his Scottish forbears had looked on in far-away Morayshire.

Outside his family circle his intimate associates were not numerous; but his friendships were intense, and his rare and concise expressions of affection were overwhelmingly strong. As I look back on this completed life, it seems to me filled with productive labors and large services from which came deep satisfactions. Grave trials and sorrows hallowed it; but its main warp and woof were both made of innumerable threads of happiness and content.

[90] In his religious convictions he was a Unitarian, and he valued highly that simple and optimistic faith; but his mind was hospitable to all forms of theological opinion, while he was strenuously averse to ecclesiasticism and aestheticism in religion. Simplicity, cheerfulness, duty, and love were the articles of his faith, and human joy and well-being their natural fruit.

 

Source: Sons of the Puritans. A Group of Brief Biographies. Boston, American Unitarian Association, 1908. Sketches reprinted from The Harvard Graduates’Magazine, Vol. VIII, No. 32 (June, 1900), pp. 469-484.

Image Source: The Harvard Graduates’Magazine, Vol. VIII, No. 32 (June, 1900), Frontspiece. Colorized by Economics in the Rear-view Mirror.

 

 

Categories
Economic History Fields Harvard Suggested Reading Syllabus

Harvard. Modern Economic History Seminar. Usher, 1937-41

For the seminar “Topics in Modern Economic History” taught by Abbott Payson Usher (1883-1965) at Harvard, I have been able to find syllabi from the fall term of 1937-38 and both terms for 1938-39 and 1940-41 which, judging from the enrollment figures reported in the annual president’s report, were the only years the seminar with Usher (alone) had actually taken place. Only after the Second World War do we see a seminar with this title having student enrollment figures reported (Usher and Walt Rostow in 1947-48 and then Usher and Gerschenkron in 1948-49.

_____________________________________________

Economics 136 (formerly 22).
Seminar.
Topics in Modern Economic History

Two consecutive hours a week, to be arranged. Professor Usher.

Source: Announcement of the Courses of Instruction offered by the Faculty of Arts and Sciences during 1937—38 (First edition). Official Register of Harvard University, Vol. 34, No. 5 (March 1, 1937), p. 149.

_____________________________________________

Enrollments in the seminar:
Topics in Modern Economic History
1937/38-1940/41

1937-38.

[Economics] 136. (formerly 22). Professor Usher.—Seminar. Topics in Modern Economic History

7 Graduates. Total 7.

 

1938-39.

[Economics] 136. Professor Usher.—Seminar. Topics in Modern Economic History

9 Graduates, 1 Senior. Total 10

 

1939-40.

[not offered]

 

1940-41.

[Economics] 136. Professor Usher.—Seminar. Topics in Modern Economic History

9 Graduates, 1 Senior. Total 10

 

Source: Harvard University. Reports of the President, 1937-38, p. 86; 1938-39, p. 99; 1940-41, p. 59.

_____________________________________________

 

[Seminar.
Topics in Modern Economic History.
A. P. Usher]

 

Calendar
Economics 136
First Term

1937-38

  1. Oct. 7.     Schools of Economic Historians.
  2. Oct. 18.   Sombart and the Ideal Type Method
  3. Nov. 5.    History as a genetic science.
  4. Nov. 19.  Historical Process: evolution or fluctuation
  5. Dec. 3.    A Pluralistic Concept of Social Evolution
  6. Dec. 17.  The Historical Significance of Locational Theory.
  7. Jan. 7     Autarchy as an objective of economic policy.

Suggested Reading

  1. Gras, N. S. B. The Rise and Development of Economic History, The Economic History Review, I, pp. 12-34.
  2. Parsons, T. Capitalism in Recent German Literature, Journal of Political Economy, vol 36, pp, 641-661; vol. 37, pp. 31-51.
  3. Sée, H. Science et Philosophie de l’histoire, pp. 116-156, 228-233.
    or
    Rickert, H. Die Grenzen der naturwissenschaftlichen Begriffsbildung, 1902, pp. 1-29, 289-304; 1913, 1-29, 256-273; 1921, pp. 1-23, 215-230.
  4. Tarde, G. Social Laws, pp. 1-67.
  5. Tarde, G. Social Laws, pp, 144-213.
    or Ogburn, W. F. Social Change, 1923. pp. 56-145.
  6. Weber, Alfred. Theory of the Location of Industries.
    Hoover, E. M. Jr. Location Theory of the Shoe and Leather Industries.
  7. Heckscher, E.F. Mercantilism, I, pp. 19-44, II, pp. 13-52.

 

Source: Harvard University Archives. Syllabi, course outlines and reading lists in Economics, 1895-2003 (HUC 8522.2.1), Box 2, Folder 1937-1938.

_____________________________________________

[1938-39 Seminar:
Topics in Modern Economic History,
A. P. Usher]

Economics 136
Reading on General Seminar Topics.

I. Schools of Economic Historians.            Oct. 4.

Gras, N. S. B. The Rise and Development of Economic History, The Economic History Review, I, pp. 12-34.
Gide, C. and Rist, C. History of Economic Thought, chapters on the historical school.

II. Sombart and the Ideal Type Method. Oct. 18.

Parsons, T. Capitalism in Recent German Literature, Journal of Political Economy, vol 36, pp. 641-661; vol. 37, pp. 31-51.

III. History as a Genetic Science    Nov. 1.

Sée, H. Science et Philosophie de l’histoire, pp. 116-156, 228-233.
or
Rickert, H. Die Grenzen der naturwissenschaftlichen Begriffsbildung, ed. 1902, pp. 1-29, 289-304; ed. 1913, 1-29, 256-273; ed. 1921, pp. 1-23, 215-230.

IV. Historical Process, evolution or fluctuation? Nov. 15.

Tarde, G. Social Laws, pp. 1-67.
Sorokin, P. Social Dynamics, I, chap. IV; vol II, ch. X.

V. A Pluralistic Concept of Social Evolution.       Nov. 29.

Tarde, G. Social Laws, pp, 144-213.
or Ogburn, W. F. Social Change (1923) pp. 56-145.

VI. Autarchy as an objective of Social Policy.      Dec. 6.

Heckscher, E.F. Mercantilism, I, pp. 19-44, II, pp. 13-52.

VII. The origin and development of liberalism. Dec. 30.

Allen, J.W. A History of Political Thought in the Sixteenth Century, (11928) Part II, ch. III; Part III, ch. VIII.
Hobhouse, L.T. Liberalism. Selections at discretion

*   *   *  *   *

[“2nd Term” in pencil]
Economics 136
1938-39

A final examination will be scheduled, consisting of an essay prepared prior to the examination upon any one of the topics listed below.

No student will be responsible for more material than is presented by the titles suggested under the various topics. Relevant material will be selected from the titles listed.

1. The significance of the scientific method for economic history.

Whitehead, A. N.       The Function of Reason,
___________________          Science and the Modern World.
___________________          The Concept of Nature.
Sée, H.            Science et Philosophie de l’histoire.

2. The problem of novelty in the theory of social evolution.

Bergson, H.    Creative Evolution.
Tarde, G.        La Logique Sociale.
___________         Social Laws.
___________         L’invention considerée comme moteur de l’evolution sociale.
Usher, A. P.    A History of Mechanical Inventions.

3. Pluralism and multilinear concepts of social evolution.

James. W.       Pluralism.
Tarde, G.        Social Laws.
___________         La Logique Sociale.
Ogburn, W. F.            Social Change.

4. The Process of Innovation.

Koffka, K.       The Growth of the Mind.
Gilfillan, S.C.   The Sociology of Invention.
Rossman, J.    The Psychology of the Inventor.
Usher, A.P.     A History of Mechanical Inventions.

5. Logical Problems of the Genetic Sciences.

Baldwin, J.M. Development and Evolution.
Rickert, H.      Die Grenzen der naturwissenschaftlichen Begriffsbildung.

6. The concept of industrial evolution.

Bücher, K. Industrial Evolution

7. The Ideal Type Method and the Theory of Social Evolution

Parsons, T. Capitalism in Recent German Literature, Journal of Political Economy, vol. 36, 641-661; 37, 31-51.
Nussbaum, F.L. A History of the Economic Institutions of Modern Europe.
or
Weber, M. General Economic History.

8. Karl Marx and the Materialistic Interpretation of History.

Bober, M. Karl Marx’s Interpretation of History.
Sée, H. The Economic Interpretation of History.

9. The Concept of Capitalism as a basis for the interpretation of modern economic history.

any one of the following texts:
Hobson, J.A.    The Evolution of Modern Capitalism.
Sée, H.           Modern Capitalism.
Hammond, J. L. and Barbara. The Rise of Modern Industry.
Bober, M. Karl Marx’s Interpretation of History.

 

Source: Harvard University Archives. Syllabi, course outlines and reading lists in Economics, 1895-2003 (HUC 8522.2.1), Box 2, Folder 1938-1939.

_____________________________________________

[1940-41 Seminar:
Topics in Modern Economic History,
A. P. Usher]

Economics 136
1940 – 1941

Calendar

  1. Oct. 3.    The Development of Economic History.
  2. Oct. 17.  Werner Sombart and Max Weber as exponents of the ideal type method
  3. Oct. 31.  History as a genetic science.
  4. Nov. 14. The Nature of Historical Process.
  5. Dec. 5.    A Pluralistic Concept of Social Evolution
  6. Dec. 19.  Mercantilism.

Reading

There will be an examination at Mid-year consisting of a three hour essay prepared on some topic drawn from, or related to, the work of the general meetings of the class. Readings suggested as preparation for class discussion will not in all instances serve as an adequate basis for the examination essay. A supplementary list of books for essay topics is therefore added.

1. The Development of Economic History.

Gide, C. and Rist, C. History of Economic Thought, Book IV, Chap. I.
Ingram, J. K. History of Political Economy. ed. 1898, chapters VI, VII; ed. 1915, chapters VI, VIII.

2. Werner Sombart and Max Weber as exponents of the ideal type method

Parsons, T. Capitalism in Recent German Literature, Journal of Political Economy, vol 36, pp. 641-661; vol. 37, pp. 31-51.
Weber, General Economic History, pp. 338-370. German ed. 1924, pp. 289-315.

3. History as a genetic science.

Sée, H. Science et Philosophie de l’histoire, pp. 116-156, 228-233.
or
Rickert, H. Die Grenzen der naturwissenschaftlichen Begriffsbildung, 1902, pp. 1-29, 289-304; 1913, 1-29, 256-273; 1921, pp. 1-23, 215-230.

4. The Nature of Historical Process.

Tarde, G.        Social Laws, pp. 1-67.

5. A Pluralistic Concept of Social Evolution

Tarde, G. Social Laws, pp, 144-213.
or Ogburn, W. F. Social Change (1923) pp. 56-145.

6. Mercantilism.

Heckscher, E.F. Mercantilism, I, pp. 19-44, II, pp. 13-52.

*   *   *  *   *

[As can be seen from the date here, for the second term of 1940-41, Usher simply used his second term syllabus from 1938-1939 with a couple of penciled additions. These additions are given below in boldface within square brackets]

Economics 136
1938-39

A final examination will be scheduled, consisting of an essay prepared prior to the examination upon any one of the topics listed below.

No student will be responsible for more material than is presented by the titles suggested under the various topics. Relevant material will be selected from the titles listed.

[In many cases, any one of the titles listed will be sufficient.]

1. The significance of the scientific method for economic history.

Whitehead, A. N.       The Function of Reason,
___________________          Science and the Modern World.
___________________          The Concept of Nature.
Sée, H.            Science et Philosophie de l’histoire.

2. The problem of novelty in the theory of social evolution.

Bergson, H.    Creative Evolution.
Tarde, G.        La Logique Sociale.
___________         Social Laws.
___________         L’invention considerée comme moteur de l’evolution sociale.
Usher, A. P.    A History of Mechanical Inventions.

3. Pluralism and multilinear concepts of social evolution.

James. W.       Pluralism.
Tarde, G.        Social Laws.
___________         La Logique Sociale.
Ogburn, W. F.            Social Change.

4. The Process of Innovation.

Koffka, K.       The Growth of the Mind.
Gilfillan, S.C.   The Sociology of Invention.
Rossman, J.    The Psychology of the Inventor.
Usher, A.P.     A History of Mechanical Inventions.

5.  Logical Problems of the Genetic Sciences.

(Baldwin, J.M. Development and Evolution.
Rickert, H.      Die Grenzen der naturwissenschaftlichen Begriffsbildung.
[Haldane, R. B. The Reign of Relativity.]

6. The concept of industrial evolution.

Bücher, K. Industrial Evolution

7. The Ideal Type Method and the Theory of Social Evolution

Parsons, T. Capitalism in Recent German Literature, Journal of Political Economy, vol. 36, 641-661; 37, 31-51.
Nussbaum, F.L. A History of the Economic Institutions of Modern Europe.
or
Weber, M. General Economic History.

8. Karl Marx and the Materialistic Interpretation of History.

Bober, M. Karl Marx’s Interpretation of History.
Sée, H. The Economic Interpretation of History.

9. The Concept of Capitalism as a basis for the interpretation of modern economic history.

any one of the following texts:
Hobson, J.A.    The Evolution of Modern Capitalism.
Sée, H.               Modern Capitalism.
Hammond, J. L. and Barbara. The Rise of Modern Industry.
Bober, M.         Karl Marx’s Interpretation of History.

[10. Religion and the Rise of Capitalism

Tawney, R. H. Religion and the Rise of Capitalism.
Robertson, H. M. The Rise of Economic Individualism.]

 

Source: Harvard University Archives. Syllabi, course outlines and reading lists in Economics, 1895-2003 (HUC 8522.2.1), Box 2, Folder 1940-41.

Image Source:  Harvard Class Album 1939.

Categories
Economists M.I.T. Regulations

MIT. Graduate Economics Program and Fellowships. 1950-51

Already by the academic year 1950-51 the M.I.T. economics department could boast seven economics professors who would still be around over a quarter of a century later, including Samuelson, Solow and Kindleberger. The printed departmental brochure along with a one-page announcement of twelve graduate fellowships, presumably sent to be posted on college and university bulletin boards, have been transcribed for this posting. Minor changes in formatting have been used to enhance readability.

 _________________________

 

Graduate Work in the Department of Economics and Social Science
Massachusetts Institute of Technology

 

THE PROGRAM

 

Our program in Economics is confined to students for the doctorate who are primarily interested in advanced study and research in

Economic Theory
Industrial Economics
Industrial Relations
International Economics
Statistics

We have an active program of continuing research in each of these fields and should like to invite a selected group of graduate students to participate with us in our explorations after they have completed their requirements for the general examinations here.

The work in Economic Theory is under the leadership of Professor Paul A. Samuelson. This includes, in addition to price analysis, the study of national income determination and business cycles. Research in these fields has been vigorous in recent years, and our objective is to train economists capable of understanding and appraising the results of this research and of adding to our empirical and theoretical understanding of these areas.

Industrial Economics, under the guidance of Professors W. Rupert Maclaurin and Max Millikan, is concerned primarily with the economic problems of the individual firm and of particular industries. The work should be enriched by the active research program now under way in the Department on “the economics of innovation,” “the process of business decisions,” and “the economics of the size of the firm.” We are anxious to have some advanced students who would like to participate in these research programs which are being worked out through “laboratory-type” collaboration of particular firms and industries.

Industrial Relations, under the leadership of Professors Charles A. Myers and Douglass V. Brown, is concerned with investigating the fundamentals of labor-management relations in modern industrial society. In addition to basic work in Economics, the program of study centers upon courses in Labor Economics, Collective Bargaining, Public Policy in Labor Relations, Personnel Administration, Social Psychology and Human Relations. A number of research projects are carried on by the Industrial Relations Section, which is a division of the Department.

Our work in International Economics is under the direction of Professors Charles P. Kindleberger and Richard M. Bissell, Jr. (who returns in June to M.I.T. from his position as Deputy Administrator of ECA). Emphasis in International economics is shared between the traditional fields of international trade and finance and that of national economic development. The training is designed to qualify the student for work in departments of government, including international institution., concerned with foreign and international economic problems. While no specialized courses are offered in the practical aspects of foreign trade, it is believed that the broad training will be regarded with increasing interest by American business concerns to aid them in the solution of their complex problems relating to foreign operations.

Instruction in Statistics, under Professor Harold Freeman, is largely centered in three areas: general theoretical statistics; probability and its foundations; modern theories of time series and prediction, particularly as applied in Economics. Some of the courses in these areas are given by the Departments of Economics and some by the Department of Mathematics. Courses are offered at elementary, intermediate, advanced and research seminar levels.

While there is ample opportunity at M.I.T. for the student interested in any one of these five fields to go as far as he wishes with his subject, there is also a common core of basic courses which the student will be expected to take in preparation for his general examinations.

We are also attempting to introduce greater realism into our program by operating a “practice school” in the summer between the first and second years of graduate study, in which we try to arrange internship experience in industry. This activity is under the guidance of Professor Paul Pigors.

For those who are going into university teaching, some pre-doctoral teaching experience will be encouraged and a considerable number of teaching fellowships will be available to students after they have completed their first year.

 

FINANCIAL ASSISTANCE TO GRADUATE STUDENTS

For the year 1950-51 we will offer up to five fellowships of $2,500, available to outstanding students in the fields mentioned above. These include the Westinghouse Educational Fellowship and the Goodyear Tire and Rubber Fellowship.

In addition, about eight fellowships and teaching scholarships will be available, ranging up to $1,600. This group includes the Clarence J. Hicks Memorial Fellowship in Industrial Relations, given by Industrial Relations Counselors, Inc., of New York.

 

REQUIREMENTS FOR ADMISSION

(a) General requirements: S.B. or A.B. degree with a good academic record from a university of recognized standing. Special emphasis will be placed on recommendations from professors or administrative officers of the college. Only students with high qualifications will be admitted.

(b) Course requirements: Three full-year college courses in social science chosen from the fields of Economics, Psychology, Sociology and History. One full-year course in college mathematics (including at least a half-year of calculus) and a full-year course in college physics are required. However, students who have had no Physics can make up this deficiency by taking a special one-semester course at the Institute. In special cases a deficiency in calculus may also be satisfied in this manner.

At the end of the second year the candidate will normally take a general examination chosen from such fields as the following: Economic Theory, Industrial economics, Economics of Innovation, Labor Economics and Labor Relations, Human Relations, Personnel administration Statistical Methods and Theory, Economic Fluctuations and Fiscal Policy, and International Economics.

Following the Institute rules the candidate for the doctor’s degree will be required to take a minor in a related filed. Possibilities include: Business Administration, History, Regional Planning, Mathematics, or any of the technical fields of specialization at the Institute in which the student is qualified to participate. Exchange arrangements between M.I.T. and Harvard University also make it possible for graduate students at either institution to take advance work at the other without extra tuition.

In addition, the candidate for the Ph.D. degree must meet the usual language and thesis requirements.

 

FURTHER INFORMATION

Those persons who are interested in learning more about the program or who wish to obtain application blanks for fellowships to aid in financing such graduate work may direct inquiries to Professor Robert L. Bishop, Department of Economics and Social Science, Massachusetts Institute of Technology, Cambridge, Massachusetts.

 

INSTRUCTING STAFF
DEPARTMENT OF ECONOMICS AND SOCIAL SCIENCE

Ralph Evans Freeman, M. A., B. Litt.
Professor of Economics; in charge of the Department

Donald Skeele Tucker, Ph.D.
Professor of Economics

William Rupert Maclaurin, D.C.S.
Professor of Economics

Norman Judson Padelford, Ph.D., LL.D.
Professor of International Relations

Paul Anthony Samuelson, Ph.D.
Professor of Economics

Richard Mervin Bissell, Jr., Ph.D.
Professor of Economics

Charles Andrew Myers, Ph.D.
Professor of Industrial Relations

Paul Pigors, Ph.D.
Associate Professor of Industrial Relations

Harold Adolph Freeman, S.B.
Associate Professor of Statistics

Charles Poor Kindleberger, Ph.D.
Associate Professor of Economics

Max Franklin Millikan, Ph.D.
Associate Professor of Economics

Alex Bavelas, Ph.D.
Associate Professor of Psychology

Robert Lyle Bishop, Ph.D.
Assistant Professor of Economics

Edgar Cary Brown, Ph.D.
Assistant Professor of Economics

Morris Albert Adelman, Ph.D.
Assistant Professor of Economics

George Pratt Shultz, Ph.D.
Assistant Professor of Industrial Relations

Robert Solow, M.A.
Assistant Professor of Statistics

Lecturer

Joseph Norbert Scanlon

Instructors

John Royston Coleman, M.A.
Stanley Martin Jacks, A.B., LL.B.
James Earnest Boyce, A.M.
Louis Cass Young, S.M.
John Lang Rawlinson, A.M.
Gilbert Koreb Krulee, S.B., M.Ed.
Roy Olton, M.A.
Herman Thomas Skofield, M.A.
Jesse Harris Proctor, Jr., M.A.

Research Associates

Robert Keen Lamb, Ph.D.
Kingman Brewster, Jr., LL.B.
Peter Robert Hofstaetter, Ph.D.

Research Assistants

William Theodore Bluhm, M.A.
Sidney Layton Smith, S.M.

Teaching Fellows

Hugh Gilbert Lovell, B.A.
Jack Dean Rogers, B.S., M.B.A.

Assistants

Ralph Haskel Bergmann, A.B.
Kenneth Alden Bohr, S.M.
Daniel Monroe Colyer, B.A.
Harold Emil Dreyer, B.S.
David Allen Eberly, S.B.
Herman Gadon, A.B.
Stuart Lee Knowlton, A.B.
Walter Sparks Measday, A.B.
Beatrice Allen Rogers, A.B., S.B.
George Joseph Strauss, B.A.

Librarian

Barbara Klingenhagen, A.B.

 _________________________

MASSACHUSETTS INSTITUTE OF TECHNOLOGY
Department of Economics and Social Science

Graduate Fellowship
1950 – 1951

 

In the year 1950-1951 M.I.T. will offer:

Up to five fellowships of $2,500 for students in the following fields:

Economic Theory
Industrial Economics
Industrial Relations
International Economics
Statistics

Up to seven fellowships with stipends up to $1,600 for specialization in these same fields.

Fellowships are available to students who wish to undertake a program of graduate work in Economics leading to the degree of doctor of philosophy. Applicants should have an A.B. or S. B. degree or anticipate the award of such a degree not later than July 1, 1950. Fellowships are awarded for one year, with possibility of renewal. They include the Westinghouse Educational Fellowship , the Goodyear Tire and Rubber Fellowship and the Clarence J. Hicks Memorial Fellowship in Industrial Relations, given by the Industrial Relations Counselors, Inc., of New York.

Fellowships are offered to those who seek career opportunities in university teaching and research, in industrial concerns in this country or abroad, in research departments of unions, and in government agencies concerned with the regulation of industry.

The Institute’s close contacts with industry, and the development within the Department of Economics and Social Science of specialized work in economic theory, the economics of innovation, industrial relations, statistics, and international economics have created a suitable environment for advanced study and research in these particular fields.

Teaching fellowships are also available; but these are normally reserved for second and third-year students.

Requests for further information or for application blanks should be addressed to Professor Robert L. Bishop, Department of Economics and Social Science, Massachusetts Institute of Technology, Cambridge, Massachusetts. Applications should be filed by March 15, 1950.

 _________________________

Source: MIT Archives. Office of the President. Records, 1930-1959. Box 77 (AC4/77), Folder 10: Economics Department 1934-49.

Image Source: MIT, Technique, 1949.

Categories
Harvard

Mathematical Economics at Harvard according to E. B. Wilson, 1936

A letter from E. B. Wilson of Harvard  to W. C. Mitchell of Columbia regarding mathematical economics at Harvard. Wilson appears to be not amused by what Schumpeter has done to the core theory course as formerly taught by Taussig. For some background see the proposal submitted by the economics department to establish courses in mathematical economics beginning 1933-34

My favourite sentence: “The fact is we are lousy with mathematical economics so near as I can make out. I suppose Leontieff leans pretty strongly that way.”

_________________________

 

HARVARD UNIVERSITY
SCHOOL OF PUBLIC HEALTH
55 SHATTUCK STREET

Boston, Massachusetts

3/27/36

 

DEPARTMENT OF VITAL STATISTICS

Edwin B. Wilson
Carl R. Doering

 

Professor Wesley C. Mitchell
161 West Twelfth Street,
New York, N.Y.

 

Dear Mitchell:

I understand Burbank plans for me to be giving next year the course on mathematical economics rather than the course on mathematical statistics which I am giving this year. This if Rollin Bennett were with us he could probably take my course.

There seems to be plenty of mathematical economics around Harvard and I judge that some people are pretty badly worried over the situation. Taussig’s great course on economic theory has now been taken over by Schumpeter and as far as I can find has been completely changed in that instead of covering a wide range of economic variables and points of view in oral discussion and trying to give the student some notion of how the theoretical economist may reason on the facts of a complicated world Schumpeter is practically giving mathematical economics. I don’t know that this is so but I am told that it is. The department is relying somewhat apparently on Chamberlin and Taylor to give general courses on economic theory without requiring much mathematics but the fact seems to be that the tradition that students should take the chief theory course which was Taussig’s and is now Schumpeter’s is so strong that practically nobody can escape Schumpeter’s course. I am told that he not only uses a great deal of mathematics, so that the course is really unintelligible to students who have not had at least two years of collegiate mathematics, but that more than this he marks the students not on how well they handle their economics in view of their total preparation but on their mathematical dexterity which makes it essential if a fellow is to have a high mark that he really be a pretty good mathematician. I have a notion that a good many members of the department are decidedly disturbed over the situation and are making some inquiries to find out whether it may not be possible for the department of mathematics to give a course more or less parallel to the one which it now gives which shall get the students further along in their mathematical notions in two years than is at present the case. You probably are aware that our layout in mathematics here is, like that elsewhere, designed primarily for students who are going to need a great deal of mathematical technique because of going into physics, or engineering, or astronomy, or mathematics itself, and isn’t ideally suited to those students who need a wide range of mathematical conceptions and a moderate range of technique and need to get it quickly as is best for students going into chemistry, physiology, economics, business, psychology, and so on. Now I happen to have repeatedly urged at Yale, at M.I.T., and informally here at Harvard where I am not technically connected with the department of mathematics, that our large institutions instead of giving one and the same course to all Freshmen and Sophomores in anything from 8 to 30 divisions should offer two parallel courses one of them directed to the students who needs to acquire a very considerable mathematical technique and covering topics selected with reference to the needs of such a person whereas the other should be differently conceived. It is a fact that an engineer with his courses on statics including strength of materials needs a pretty thorough grounding in trigonometric analysis. It is also important in the study of alternating current machinery. He may also need a considerable amount of analytical geometry if he is to be at home with ellipsoids of inertia and with stress and strain relations in the theory of materials. He must have considerable familiarity with the integration of differential equations. He really needs two and a half years of continuous and hard mathematics. Now for the other people whom I mentioned trigonometry is a matter that can be covered in very few exercises. They don’t have to do surveying, they don’t have to solve triangles, they don’t have to do analytical statics. On the other hand they do need rather more algebra, including choice and chance, for which there is no special necessity on the part of the engineer or physicist, at any rate at an early stage and they do need to get through with their mathematics say in a year and a half instead of two, and a half years. It is all perfectly simple to do in all institutions large enough so that there is no additional cost in running two parallel courses instead of a single course but I don’t know of any institution that does it. If I were arranging the courses on mathematics for the Freshman and Sophomore years in our major institutions there would be much greater variety in the offering, but this is beside the point. The fact is we are lousy with mathematical economics so near as I can make out. I suppose Leontieff leans pretty strongly that way.

I am sorry you have so much administrative work in the National Bureau. I wish you could be let alone to do your research. For that matter I wish I could. I am in too many things and this job for the National Resources Committee has been a great burden to me this year. Then Gay resigned from the Executive Committee on the Tercentenary and I was put in his place. The president has put me on his project committee to represent sociology as near as I can make out. My time has been dissipated. Some of the things that I was anxious to do when I gave up the presidency of the SSRC I have done but there still remain to be done a great many of those things which I had every reason to believe would have been finished by this time. In the meantime I suppose I have done a few other odds and ends that weren’t on my program because my position here is essentially a consulting position and I have to take in and think about problems which have been initiated by other persons but which can’t be wound up by them because they haven’t adequate knowledge of statistical methods and I fear even not of logical methods.

I wish the SSRC this summer would go up to the South Shore or the North Shore so that their members would be near at hand for the Tercentenary Conference of the social sciences. It would seem to me that the Symposium on Factors Affecting Human Behavior and that on the Relations of Authority to the Individual and that on Cultural Diffusion which occupy the 5 days from September 7th to September 11th inclusive would be right up the main track of the SSRC and that we would be likely to have more of the people here if the Council met up in this vicinity. I daresay that wherever it meets I shan’t have much time for it because of pressure of work on the small committee consisting of Shapley, Henderson, Nock and myself with Jerome Greene which is to take care of the Academic end of this large conference.

Yours very sincerely,
[signed: E. B. Wilson]

 

Source: Columbia University Libraries Manuscript Collections. Mitchell, W. C. Collection, Box 14 (Correspondence Ve—Z), Folder “Wilson, Edwin B., Boston, 27 March 1936, To Wesley C. Mitchell”.